COGNITION

Pataasin ang iyong marka sa homework at exams ngayon gamit ang Quizwiz!

When assessing an older adult, the nurse anticipates an increase in which component of respiratory status? A) Vital capacity B) Gas exchange and diffusing capacity C) Residual lung volume D) Cough efficiency

Residual lung volume

Which would be an appropriate intervention for a client experiencing an anxiety attack?

Staying with the client and speaking in short sentences

A nurse is assessing a client brought to the emergency room by his daughter. Which statement by the daughter would most likely lead the nurse to suspect that the client may have an infection?

"All of a sudden my dad seemed to become confused."

A client asks the nurse, "How can I tell if what I am experiencing is just regular worrying and not an anxiety disorder?" What is the nurse's best response?

"If you are unable to function occupationally and socially because of the anxiety"

A nurse is caring for a client with dementia. A family member of the client asks what the most common cause of dementia is. Which response by the nurse is most appropriate?

"The most common cause of dementia in the elderly is Alzheimer's disease."

A client with Guillain-Barré syndrome has paralysis affecting the respiratory muscles and requires mechanical ventilation. When the client asks the nurse about the paralysis, how should the nurse respond?

"The paralysis caused by this disease is temporary."

Approximately what percentage of home care required by elderly people with Alzheimer disease is provided by informal caregivers? 80 90 70 60

80

A client with myasthenia gravis is admitted with an exacerbation. The nurse is educating the client about plasmapherisis and explains this in which of the following statements?

Antibodies are removed from the plasma.

Which of the following is the priority nursing intervention for a patient in myasthenic crisis?

Assessing respiratory effort

A patient diagnosed with MS 2 years ago has been admitted to the hospital with another relapse. The previous relapse followed a complete recovery with the exception of occasional vertigo. What type of MS does the nurse recognize the patient most likely has? A. benign B. primary progressive C. relapsing-remitting (RR) D. disabling

C

The nurse caring for a patient diagnosed with Guillain-Barré syndrome is planning care with regard to the clinical manifestations associated this syndrome. The nurse's communication with the patient should reflect the possibility of what sign or symptom of the disease? A) Intermittent hearing loss B) Tinnitus C) Tongue enlargement D) Vocal paralysis

D) Vocal paralysis

Which is an age-related change in the respiratory system? A) Decreased gas exchange B) Loss of muscle strength and size C) Difficulty swallowing D) Increased blood pressure

Decreased gas exchange

Which factor alters urinary elimination patterns in older adults?

Decreased muscle tone

Which condition is characterized by a decline in intellectual functioning?

Dementia

Impaired balance and uncontrolled tremors of Parkinson's disease is correlated with which neurotransmitter?

Dopamine pg. 2063

An acoustic neuroma is a benign tumor of which cranial nerve?

Eighth pg. 2053

A nurse is teaching nursing assistants in an extended-care facility measures to protect the skin of elderly clients. Which of the following measures is the nurse likely to recommend?

Encouraging clients to avoid cigarette smoking

Which of the following is a cause of a calcium renal stone?

Excessive intake of vitamin D

In a client with benign prostatic hyperplasia (BPH), which assessment finding provides the best indication of urinary retention?

Frequency

A nurse is preparing to assess an older adult patient. Which approach would the nurse most likely use? Body system assessment Functional assessment Head-to-toe assessment Body region assessment

Functional assessment

A 37-year-old mother of three has just been diagnosed with a grade I meningioma. As part of patient education, the nurse tells the patient that:

Growth is slow and symptoms are caused by compression rather than tissue invasion pg. 2053

A patient is admitted complaining of low back pain. What will best assist the nurse in determining if the pain is related to a herniated lumbar disc?

Have the patient lie on his back and lift his leg, keeping it straight.

A client is admitted to undergo lumbar laminectomy for treatment of a herniated disk. Which action should the nurse take first to promote comfort preoperatively?

Help the client assume a more comfortable position.

Which of the following is the most common cause of acute encephalitis in the United States?

Herpes simplex virus (HSV)

Which disease is a chronic, progressive, hereditary disease of the nervous system that results in progressive, involuntary dancelike movements and dementia?

Huntington disease pg. 2069

A client in a nursing home is diagnosed with Alzheimer's disease. He exhibits the following symptoms: difficulty with recent and remote memory, irritability, depression, restlessness, difficulty swallowing, and occasional incontinence. This client is in what stage of Alzheimer's disease? A) IV B) III C) I D) II

II

A patient with a brain tumor is complaining of headaches that are worse in the morning. What does the nurse know could be the reason for the morning headaches?

Increased intracranial pressure

Which nursing diagnosis takes the highest priority for a client with parkinsonian crisis?

Ineffective airway clearance

A male client who has undergone a cervical discectomy is being discharged with a cervical collar. Which of the following would be most appropriate to include the client's discharge plan?

Keeping the head in a neutral position

Examination of a client's bladder stones reveal that they are primarily composed of uric acid. The nurse would expect to provide the client with which type of diet?

Low purine

A client with spinal cord compression from a tumor must undergo diagnostic testing. Which of the following is the most likely procedure for this client?

Magnetic resonance imaging pg. 2056

What nursing intervention will best assist the patient with chorea?

Monitor the patient on bed rest.

The nurse is performing an initial nursing assessment on a client with possible Guillain-Barre syndrome. Which of the following findings would be most consistent with this diagnosis?

Muscle weakness and hyporeflexia of the lower extremities

The nurse teaches the patient with which disorder his or her disease is due to decreased levels of dopamine in the basal ganglia of the brain?

Parkinson's disease

A client with moderate Alzheimer's disease has been eating poorly, losing weight, and playing with food at meals. The nurse best intervenes by

Placing one food at a time in front of the client during meals

Which action is included in a nurse's role when providing home care for a client with Alzheimer disease?

Provide emotional and physical support

Which of the following statements indicate appropriate nursing intervention for a patient with postpolio syndrome?

Providing care aimed at slowing the loss of strength and maintaining overall well-being.

Which of the following statements reflect nursing interventions of a patient with post-polio syndrome?

Providing care aimed at slowing the loss of strength and maintaining the physical, psychological and social well being of the patient

The nurse teaches the patient that corticosteroids will be used to treat his brain tumor for which reason?

Reduce cerebral edema

Which of the following provides the best outcome for most tumor types?

Surgery

A client who experiences panic anxiety around dogs is sitting in a room with a dog and the client's nurse therapist. The nurse therapist is using which behavioral intervention for this type of anxiety?

Systematic desensitization

Which actions by the nurse will assist in promoting an older adult's adherence to medication therapy? Select all that apply. Educate the client to keep all medications and bottles for future reference. Use easy-to-open lids. Encourage the patient to use multiple pharmacies to obtain cheapest prices. Provide a written medication schedule. Encourage the client to keep a list of medications and review it frequently for updates. Instruct the client not to take herbal supplements.

Use easy-to-open lids. Provide a written medication schedule. Encourage the client to keep a list of medications and review it frequently for updates.

Which factor should be checked when evaluating the effectiveness of an alpha-adrenergic blocker given to a client with benign prostatic hyperplasia (BPH)?

Voiding pattern

A patient has been diagnosed with meningococcal meningitis at a community living home. When should prophylactic therapy begin for those who have had close contact with the patient?

Within 24 hours after exposure

Medical management of arthropod-borne virus (arboviral) encephalitis is aimed at

controlling seizures and increased intracranial pressure.

The most common affective or mood disorder of old age is

depression.

Which basic of client care, occurring during the acute phase, is most helpful in promoting the rehabilitation of a client following a debilitating cerebrovascular accident?

Prevention of joint contractures

A client has a herniated disk in the region of the third and fourth lumbar vertebrae. Which nursing assessment finding most supports this diagnosis?

Severe lower back pain

A 68-year-old retired salesman was brought by squad into the acute care facility where you practice nursing. His wife accompanies him and relates how the client reported a severe headache, shortly after he was unable to talk or move his right arm and leg. His wife indicates the client has hypertension. What should be your focus of management during this phase?

Preventing further neurologic damage

Which drug should be available to counteract the effect of Tensilon?

Atropine

A 35-year-old woman is diagnosed with a peripheral neuropathy. When making her plan of care, the nurse knows to include what in patient teaching? Select all that apply. A) Inspect the lower extremities for skin breakdown. B) Footwear needs to be accurately sized. C) Immediate family members should be screened for the disease. D) Assistive devices may be needed to reduce the risk of falls. E) Dietary modifications are likely necessary.

A) Inspect the lower extremities for skin breakdown. B) Footwear needs to be accurately sized. D) Assistive devices may be needed to reduce the risk of falls.

A male patient with a metastatic brain tumor is having a generalized seizure and begins vomiting. What should the nurse do first? A) Perform oral suctioning. B) Page the physician. C) Insert a tongue depressor into the patient's mouth. D) Turn the patient on his side.

Ans: D Feedback: The nurse's first response should be to place the patient on his side to prevent him from aspirating emesis. Inserting something into the seizing patient's mouth is no longer part of a seizure protocol. Obtaining supplies to suction the patient would be a delegated task. Paging or calling the physician would only be necessary if this is the patient's first seizure.

Which of the following schedules would be most appropriate for the care of a 28-year-old female hospitalized with a diagnosis of myasthenia gravis? A) All at one time, to provide a longer rest period B) Before meals, to stimulate her appetite C) In the morning, with frequent rest periods D) Before bedtime, to promote rest

Ans: C Feedback: Myasthenia gravis is characterized by extreme muscle weakness, which generally worsens after effort and improves with rest. The schedule for procedures should be spaced to allow for rest in between. Procedures should be avoided before meals, or the patient may be too exhausted to eat. Procedures should also be avoided at bedtime.

A client with Parkinson's disease is prescribed levodopa (L-dopa) therapy. Improvement in which area indicates effective therapy? a) alertness b) appetite c) mood d) muscle rigidity

D) muscle rigidity Explanation: Levodopa is prescribed to decrease severe muscle rigidity. Levodopa does not improve mood, appetite, or alertness in a client with Parkinson's disease

Which action by the nurse demonstrates ageism

Directing all health decisions to the older adult's child

Which of the following is a late symptom of spinal cord compression?

Paralysis

A client is scheduled for a laminectomy to repair a herniated intervertebral disk. When developing the postoperative care plan, the nurse should include which action?

Turning the client from side to side, using the logroll technique

A client is to undergo a TURP for BPH. Which statement is accurate with regard to a TURP?

Urethral strictures are more frequent for TURP than for nontransurethral procedures.

The nurse advises a patient with renal stones to avoid eating shellfish, asparagus, and organ meats. She emphasizes these foods because she knows that his renal stones are composed of which of the following substances?

Uric acid

A client with a neurologic impairment reports to his home care nurse that he has been having problems with constipation. Which of the following foods might the nurse recommend?

Vegetables

Oncotic pressure refers to the...

osmotic pressure exerted by proteins.

A client is scheduled for a transsphenoidal hypophysectomy to remove a pituitary tumor. Preoperatively, the nurse should assess for potential complications by:

performing capillary glucose testing every 4 hours. pg. 1880

A client with cerebral metastasis suddenly experiences a seizure for which phenytoin 10 mg/kg intravenously is ordered as an initial loading dose. The client weighs 132 pounds. How many milligrams of phenytoin should the client receive? Enter the number ONLY.

600

Which would be included as a condition that can cause an elevated PSA level in the absence of prostate cancer? Select all that apply.

Acute urinary retention

The priority concern for people with mood disorders is what?

Safety

Which of the following terms is used to describe rapid, jerky, involuntary, and purposeless movements of the extremities?

Chorea

Which action by the nurse demonstrates ageism? A) Providing the same high quality of care to all clients B) Directing all health decisions to the older adult's child C) Allowing adequate time for the older adult to complete tasks D) Encouraging the older adult to develop routines not associated with work

Directing all health decisions to the older adult's child

Which of the following should be included when teaching a client about the management of benign prostatic hyperplasia (BPH)? Select all that apply.

Do not delay the urge to void. Schedule digital rectal exams.

A client has recently brought her elderly mother home to live with her family. The client states that her mother has moderate Alzheimer's disease and asks about appropriate activities for her mother. The nurse tells the client to... A) Ensure that the mother does not have access to car keys or drive an automobile. B) Turn off lights at night so that the mother differentiates night and day. C) Encourage the mother to take responsibility for cooking and cleaning the house. D) Allow the mother to smoke cigarettes outside on the porch without supervision.

Ensure that the mother does not have access to car keys or drive an automobile.

The patient with a brain tumor may be at increased risk for aspiration. The most important nursing intervention includes which of the following.

Evaluation of gag reflex and ability to swallow

An elderly client is hospitalized for treatment related to leukemia. Family members want to visit with a toddler who has a cold. It would be best for the nurse to... Inform the family to either wash their hands or use the hand sanitizer. Instruct the family to remove the toddler from the room for the protection of the client. Allow the toddler to remain in the room if a family member wipes the toddler's nose. Ask the family to leave the client's room.

Instruct the family to remove the toddler from the room for the protection of the client

An elderly client reports that he feels like he voids frequently during the day and at night but cannot empty his bladder. The nurse instructs the client to

Limit ingestion of caffeinated beverages

The client's lab values are sodium 166 mEq/L, potassium 5.0 mEq/L, chloride 115 mEq/L, and bicarbonate 35 mEq/L. What condition is this client likely to have, judging by anion gap? Metabolic acidosis Respiratory alkalosis Metabolic alkalosis Respiratory acidosis

Metabolic acidosis

A client with muscle weakness and an abnormal gait is being evaluated for muscular dystrophy. Which test or finding confirms muscular dystrophy?

Muscle biopsy

The most common cause of cholinergic crisis includes which of the following?

Overmedication

An older adult female patient tells the nurse, "I have lost an inch [2.5 cm] of height and have a hump on my back. What can I do about this?" What is the best response by the nurse? "In order to prevent further bone loss, eat a diet high in calcium and low in phosphorus." "Supplement your diet with a multivitamin." "You can reverse the bone loss with surgical intervention." "In order to prevent further bone loss, eat a diet high in magnesium and high in phosphorus."

"In order to prevent further bone loss, eat a diet high in calcium and low in phosphorus."

A home care nurse makes a visit to a client with Parkinson's disease who is being cared for by his spouse. During the visit, the spouse says, "I'm just so tired. I have to do just about everything for him." Which response by the nurse would be most appropriate?

"You sound a bit overwhelmed. Tell me more about what's happening."

A patient with Parkinson's disease asks the nurse what can be done to prevent problems with bowel elimination. What would be an intervention that would assist this patient with a regular stool pattern?

Adopt a diet with moderate fiber intake pg.

The nurse caring for a patient diagnosed with Parkinson's disease has prepared a plan of care that would include what goal? A) Promoting effective communication B) Controlling diarrhea C) Preventing cognitive decline D) Managing choreiform movements

Ans: A Feedback: The goals for the patient may include improving functional mobility, maintaining independence in ADLs, achieving adequate bowel elimination, attaining and maintaining acceptable nutritional status, achieving effective communication, and developing positive coping mechanisms. Constipation is more likely than diarrhea and cognition largely remains intact. Choreiform movements are related to Huntington disease.

A patient with suspected Parkinson's disease is initially being assessed by the nurse. When is the best time to assess for the presence of a tremor? A) When the patient is resting B) When the patient is ambulating C) When the patient is preparing his or her meal tray to eat D) When the patient is participating in occupational therapy

Ans: A Feedback: The tremor is present while the patient is at rest; it increases when the patient is walking, concentrating, or feeling anxious. Resting tremor characteristically disappears with purposeful movement, but is evident when the extremities are motionless. Consequently, the nurse should assess for the presence of a tremor when the patient is not performing deliberate actions.

A patient with Parkinson's disease is experiencing episodes of constipation that are becoming increasingly frequent and severe. The patient states that he has been achieving relief for the past few weeks by using OTC laxatives. How should the nurse respond? A) "It's important to drink plenty of fluids while you're taking laxatives." B) "Make sure that you supplement your laxatives with a nutritious diet." C) "Let's explore other options, because laxatives can have side effects and create dependency." D) "You should ideally be using herbal remedies rather than medications to promote bowel function."

Ans: C Feedback: Laxatives should be avoided in patients with Parkinson's disease due to the risk of adverse effects and dependence. Herbal bowel remedies are not necessarily less risky.

A patient, diagnosed with cancer of the lung, has just been told he has metastases to the brain. What change in health status would the nurse attribute to the patient's metastatic brain disease? A) Chronic pain B) Respiratory distress C) Fixed pupils D) Personality changes

Ans: D Feedback: Neurologic signs and symptoms include headache, gait disturbances, visual impairment, personality changes, altered mentation (memory loss and confusion), focal weakness, paralysis, aphasia, and seizures. Pain, respiratory distress, and fixed pupils are not among the more common neurologic signs and symptoms of metastatic brain disease.

A 37-year-old man is brought to the clinic by his wife because he is experiencing loss of motor function and sensation. The physician suspects the patient has a spinal cord tumor and hospitalizes him for diagnostic testing. In light of the need to diagnose spinal cord compression from a tumor, the nurse will most likely prepare the patient for what test? A) Anterior-posterior x-ray B) Ultrasound C) Lumbar puncture D) MRI

Ans: D Feedback: The MRI scan is the most commonly used diagnostic procedure. It is the most sensitive diagnostic tool that is particularly helpful in detecting epidural spinal cord compression and vertebral bone metastases.

Bell's palsy is a disorder of cranial nerve VII. What are the clinical manifestations of the disorder? A) Tinnitus B) Facial paralysis C) Pain at the base of the tongue D) Diplopia

Ans: B Feedback: Bell's palsy is characterized by facial dysfunction, weakness, and paralysis.

The nurse assessing a patient with multiple sclerosis understands that due to the pathophysiology of this disease process which of the following is the expected primary finding on the MRI? A) Subarachnoid hemorrhage B) Presence of multiple plaques C) Atrophy of the caudate nuclei D) Presence of a tumor

Ans: B Feedback: MRI is the primary diagnostic tool for visualizing plaques, documenting disease activity, and evaluating the effect of treatment. A subarachnoid hemorrhage would be seen on an MRI from a ruptured aneurysm. Atrophy of the caudate nuclei is seen in Huntington's disease. The presence of a tumor indicates brain tumor.

A hospitalized older adult complains of increased coughing and shortness of breath. The nurse assesses the vital signs as temperature 100.2°F oral, respirations 18, pulse 88, and BP 128/80. What action should the nurse do next?

Assess lung sounds and sputum.

An elderly client is contemplating a move to a continuing care retirement community (CCRC). The nurse assesses that the client requires assistance with food preparation and recommends placement in

Assisted-living apartment

An elderly client is contemplating a move to a continuing care retirement community (CCRC). The nurse assesses that the client requires assistance with food preparation and recommends placement in... A) Acute care facilty B) Assisted-living apartment C) Independent dwelling D) Skilled nursing facility

Assisted-living apartment

A patient suspected of having GBS has had a lumbar puncture for cerebrospinal fluid (CSF) evaluation. When reviewing the laboratory results, what does the nurse find that is diagnostic for this disease? A. glucose in the CSF B. elevated protein levels in the CSF C. red blood cells present in the CSF D. white blood cells in the CSF

B

Which is an expected outcome for a client with Parkinson's disease who has had a pallidotomy? a) reduced emotional stress b) improved functional ability c) better appetite d) increased alertness

B) improved functional ability Explanation: The goal of a pallidotomy is to improve functional ability for the client with Parkinson's disease. This is a priority. The pallidotomy creates lesions in the globus pallidus to control extrapyramidal disorders that affect control of movement and gait. If functional ability is improved by the pallidotomy, the client may experience a secondary response of an improved emotional response, but this is not the primary goal of the surgical procedure. The procedure will not improve alertness or appetite.

The nurse is discharging a patient home after surgery for trigeminal neuralgia. What advice should the nurse provide to this patient in order to reduce the risk of injury? A) Avoid watching television or using a computer for more than 1 hour at a time. B) Use OTC antibiotic eye drops for at least 14 days. C) Avoid rubbing the eye on the affected side of the face. D) Rinse the eye on the affected side with normal saline daily for 1 week.

C) Avoid rubbing the eye on the affected side of the face.

A nurse is obtaining the health history of a 72-year-old woman who has come to the ambulatory care center for an evaluation. When obtaining information about the woman's sleep patterns, which of the following would the nurse expect to assess? A) Complaints about frequently waking up during the night B) Statements that she rarely takes naps during the day C) Reports that she has trouble waking up from sleeping D) Reports that she falls asleep more quickly

Complaints about frequently waking up during the night

Which older adult is at highest risk for medication-related toxicity? A) A 86-year-old who has had Type-2 diabetes for the past 6 years B) A 40-year-old with severe diarrhea C) A 56-year old with gastric ulcer disease D) A 65-year-old with renal insufficiency

D) A 65-year-old with renal insufficiency

The nurse is preparing to provide care for a patient diagnosed with myasthenia gravis. The nurse should know that the signs and symptoms of the disease are the result of what? A) Genetic dysfunction B) Upper and lower motor neuron lesions C) Decreased conduction of impulses in an upper motor neuron lesion D) A lower motor neuron lesion

D) A lower motor neuron lesion

A middle-aged woman has sought care from her primary care provider and undergone diagnostic testing that has resulted in a diagnosis of MS. What sign or symptom is most likely to have prompted the woman to seek care? A) Cognitive declines B) Personality changes C) Contractures D) Difficulty in coordination

D) Difficulty in coordination

During an interview, the nurse has asked a client with depression about any hopes or plans for the future. In response, the client silently made a gesture of drawing the index finger from one side of the client's throat to the other. The nurse has informed the client that this must be communicated to the care team. What is the main rationale for the nurse's action?

Ensuring the client's safety

An 89-year-old retired government clerk is being admitted to your rehabilitation hospital as a result of the tetraplegia caused by stroke. Her condition is stable, and after admission, she will begin physical and psychological therapy. An important part of your nursing management is to reposition the client every 2 hours. What is the rationale behind this intervention?

Maintain sufficient integument capillary pressure

A nurse is assessing a patient's urinary output as an indicator of diabetes insipidus. The nurse knows that an hourly output of what volume over 2 hours may be a positive indicator? a) 100 to 150 mL/h b) 50 to 100 mL/h c) More than 200 mL/h d) 150 to 200 mL/h

More than 200 mL/h Explanation: For patients undergoing dehydrating procedures, vital signs, including blood pressure, must be monitored to assess fluid volume status. An indwelling urinary catheter is inserted to permit assessment of renal function and fluid status. During the acute phase, urine output is monitored hourly. An output greater than 200 mL per hour for 2 consecutive hours may indicate the onset of diabetes insipidus (Hickey, 2009).

An elderly client recovering from a hip repair becomes disoriented and tries to get out of bed frequently. The client states, "I forget I am in the hospital." The best nursing intervention is to Administer an oral dose of prescribed alprazolam (Xanax). Post a sign stating "You are in the hospital" at the client's eye level. Place the client in a Posey chest restraint with ties attached to the bed frame. Raise the upper and lower side rails of the bed.

Post a sign stating "You are in the hospital" at the client's eye level.

A woman comes to her health care provider's office with signs and symptoms of kidney stones. Which of the following should be the primary medical management goal?

Relieve the pain.

A nurse is reviewing a journal article about benign prostatic hyperplasia and possible risk factors associated with this condition. Which factor would the nurse most likely find as playing a role in increasing a man's risk for this condition? Select all that apply.

Smoking Hypertension Diabetes

The nurse brings the older adult patient a dinner tray and observes the patient placing excess amounts of salt on the food. What suggestions for flavoring can the nurse provide to decrease the amount of salt the patient is placing on her food? (Select all that apply.) Use pepper instead of salt. Use lemon instead of salt to flavor food. Use low-sodium herbs and spices. Drink water before the meal. Use an alcohol-based mouthwash prior to eating.

Use pepper instead of salt. Use lemon instead of salt to flavor food. Use low-sodium herbs and spices.

A client at an extended-care facilty who has Alzheimer's disease is awake throughout the night. The nurse intervenes with activities that will promote sleep at night, which include

Walking the client in the facility yard during the day

To encourage adequate nutritional intake for a client with Alzheimer's disease, a nurse should:

stay with the client and encourage him to eat.

The daughter of a patient with Huntington's disease asks the nurse what the risk is of her inheriting the disease. What is the best response by the nurse?

"If one parent has the disorder, there is a 50% chance that you will inherit the disease." pg. 2069

Which of the following is the primary medical management of arthropod-borne virus (arboviral) encephalitis?

"You must avoid stress and extreme fatigue, because these can trigger a relapse."

Several staff members are taking a break in the unit's conference room when one of them states, "I dread getting old and having to retire. I don't want to just sit on the porch in my rocking chair." The statement reflects which of the following? Chronological aging Geriatrics Ageism Gerontology

Ageism

A patient has just returned to the unit from the PACU after surgery for a tumor within the spine. The patient complains of pain. When positioning the patient for comfort and to reduce injury to the surgical site, the nurse will position to patient in what position? A) In the high Fowler's position B) In a flat side-lying position C) In the Trendelenberg position D) In the reverse Trendelenberg position

Ans: B Feedback: After spinal surgery, the bed is usually kept flat initially. The side-lying position is usually the most comfortable because this position imposes the least pressure on the surgical site. The Fowler's position, Trendelenberg position, and reverse Trendelenberg position are inappropriate for this patient because they would result in increased pain and complications.

A patient, brought to the clinic by his wife and son, is diagnosed with Huntington disease. When providing anticipatory guidance, the nurse should address the future possibility of what effect of Huntington disease? A) Metastasis B) Risk for stroke C) Emotional and personality changes D) Pathologic bone fractures

Ans: C Feedback: Huntington disease causes profound changes to personality and behavior. It is a nonmalignant disease and stroke is not a central risk. The disease is not associated with pathologic bone fractures.

Which of the following primary manifestations is the nurse most likely to assess in a patient diagnosed with MS? A) Dementia B) Bradykinesia C) Contracture deformities D) Difficulty in coordination

Ans: D Feedback: The primary symptoms most commonly reported with patients who have MS are difficulties with coordination, spasticity of the extremities, and loss of coordination. Secondary symptoms of MS include contracture deformities and rarely dementia.

Which is an age-related change in the respiratory system? A) Decreased gas exchange B) Increased blood pressure C) Difficulty swallowing D) Loss of muscle strength and size

Decreased gas exchange

The nurse is expecting to admit a client with a diagnosis of meningitis. While preparing the client's room, which of the following would the nurse most likely have available?

Equipment to maintain infection control precautions

Older adults, who are more subject to falls, may fracture one or more ribs and be more susceptible to which condition after a rib fracture?

Pneumonia

The overall goals of care for individuals experiencing a stress response are to focus on interventions to develop ...

positive coping skills.

A client with Alzheimer's disease is being treated for malnutrition and dehydration. The nurse decides to place him closer to the nurses' station because of his tendency to: A) not change his position often. B) exhibit acquiescent behavior. C) wander. D) forget to eat.

wander.

A family of a patient with Alzheimer's disease asks the nurse what causes this condition? Which response by the nurse would be most appropriate? "A specific gene is involved in the development of this disorder." "Evidence shows that there are changes in nerve cells and brain chemicals." "This condition is most likely due to a stroke that the patient didn't realize he had." "The numerous drugs that he was taking contributed to his current confusion."

"Evidence shows that there are changes in nerve cells and brain chemicals."

During a night shift, a hospitalized client with depression tells a nurse that the client is going to kill himself or herself. The client is placed on constant observation. When the client asks to use the toilet, the nurse follows the client into the bathroom. The client says, "You don't need to follow me into the bathroom. Give me some space." Which response by the nurse is most appropriate?

"I must stay with you until we are sure you will not hurt yourself."

The nurse in an extended care facility is planning the daily activities of a patient with postpolio syndrome. The nurse recognizes the patient will best benefit from physical therapy when it is scheduled at what time? A) Immediately after meals B) In the morning C) Before bedtime D) In the early evening

Ans: B Feedback: Important activities for patients with postpolio syndrome should be planned for the morning, as fatigue often increases in the afternoon and evening.

A patient with Guillain-Barre' has had arterial blood gases (ABGs) drawn. Which of the following ABG values indicates that the patient's status is deteriorating? A) pH 7.37 B) PaCO2 60 C) HCO3 24 D) Oxygen saturation of 94%

Ans: B Feedback: A PaCO2 of 60 places the patient with Guillain-Barre' in an acidotic state due to hypoventilation from respiratory muscle weakness. The pH, HCO3, and oxygen saturation are within normal levels.

The nurse assesses for euphoria in a client with multiple sclerosis, looking for what characteristic clinical manifestation? a) an exaggerated sense of well-being b) slurring of words when excited c) visual hallucinations d) inappropriate laughter

A) an exaggerated sense of well-being Explanation: A client with multiple sclerosis may have a sense of optimism and euphoria, particularly during remissions. Euphoria is characterized by mood elevation with an exaggerated sense of well-being. Inappropriate laughter, slurring of words, and visual hallucinations are uncharacteristic of euphoria

A male patient presents at the free clinic with complaints of impotency. Upon physical examination, the nurse practitioner notes the presence of hypogonadism. What diagnosis should the nurse suspect? A) Prolactinoma B) Angioma C) Glioma D) Adrenocorticotropic hormone (ACTH)-producing adenoma

Ans: A Feedback: Male patients with prolactinomas may present with impotence and hypogonadism. An ACTH-producing adenoma would cause acromegaly. The scenario contains insufficient information to know if the tumor is an angioma, glioma, or neuroma.

A patient with Parkinson's disease is undergoing a swallowing assessment because she has recently developed adventitious lung sounds. The patient's nutritional needs should be met by what method? A) Total parenteral nutrition (TPN) B) Provision of a low-residue diet C) Semisolid food with thick liquids D) Minced foods and a fluid restriction

Ans: C Feedback: A semisolid diet with thick liquids is easier for a patient with swallowing difficulties to consume than is a solid diet. Low-residue foods and fluid restriction are unnecessary and counterproductive to the patient's nutritional status. The patient's status does not warrant TPN.

A nurse is caring for an older adult client with advanced Parkinson's disease. Which client statement about advance directives indicates a need for further instruction? a) "My family will take care of me. I've given my daughter durable power of attorney for health care." b) "I don't really need to sign anything. I'm depending on my physician to tell my family what to do if something bad happens." c) "I've signed the advance directive papers and will fight to maintain the highest quality of life until my time comes." d) "I know that I'll eventually be unable to make decisions. Signing an advance directive now will save my family grief."

B) "I don't really need to sign anything. I'm depending on my physician to tell my family what to do if something bad happens." Explanation: The client requires additional teaching if the client states that he/she will depend on the physician to tell the family what to do in regards to his/her health. The client should not rely on the physician to tell the family what to do. The best way for the client to convey his/her health care wishes is to put them in writing in an advance directive. The client stating that he/she has designated his/her daughter to make health care decisions when the client cannot, that the client has signed an advance directive, or that the client knows an advance directive will help when he/she is unable to make decisions indicate that the client has made decisions about his/her end-of-life care.

A male patient presents to the clinic complaining of a headache. The nurse notes that the patient is guarding his neck and tells the nurse that he has stiffness in the neck area. The nurse suspects the patient may have meningitis. What is another well-recognized sign of this infection? A) Negative Brudzinski's sign B) Positive Kernig's sign C) Hyperpatellar reflex D) Sluggish pupil reaction

B) Positive Kernig's sign

The nurse is caring for a patient with multiple sclerosis (MS). The patient tells the nurse the hardest thing to deal with is the fatigue. When teaching the patient how to reduce fatigue, what action should the nurse suggest? A) Taking a hot bath at least once daily B) Resting in an air-conditioned room whenever possible C) Increasing the dose of muscle relaxants D) Avoiding naps during the day

B) Resting in an air-conditioned room whenever possible

A nurse is teaching a client with multiple sclerosis (MS). When teaching the client how to reduce fatigue, the nurse should tell the client to: a) increase the dose of muscle relaxants. b) take a hot bath. c) rest in an air-conditioned room. d) avoid naps during the day.

C) rest in an air-conditioned room. Explanation: Fatigue is a common symptom in clients with MS. Lowering the body temperature by resting in an air-conditioned room may relieve fatigue; however, extreme cold should be avoided. A hot bath or shower can increase body temperature, producing fatigue. Muscle relaxants, ordered to reduce spasticity, can cause drowsiness and fatigue. Frequent rest periods and naps can relieve fatigue. Other measures to reduce fatigue in the client with MS include treating depression, using occupational therapy to learn energy-conservation techniques, and reducing spasticity

An elderly client exhibits blood pressure of 110/76 while prone, 100/72 sitting, and 92/64 standing. The nurse instructs the client to... A) Minimize the use of stool softeners. B) Take daily hot showers. C) Use whirlpool baths for relaxation. D) Ingest five or six small meals each day.

D) Ingest five or six small meals each day.

A patient is exhibiting bradykinesia, rigidity, and tremors related to Parkinson's disease. The nurse understands that these symptoms are directly related to what decreased neurotransmitter level?

Dopamine

The nurse is caring for a client with a traumatic brain injury who has developed increased intracranial pressure resulting in syndrome of inappropriate antidiuretic hormone (SIADH). While assessing this client, the nurse expects which of the following findings? a) Excessive urine output and serum hyponatremia b) Excessive urine output and decreased urine osmolality c) Oliguria and serum hyponatremia d) Oliguria and serum hyperosmolarity

Oliguria and serum hyponatremia Correct Explanation: SIADH is the result of increased secretion of antidiuretic hormone (ADH). The client becomes volume overloaded, urine output diminishes, and serum sodium concentration becomes dilute. (less)

A patient is diagnosed with an aggressive, primary malignant brain tumor. The nurse is aware that the glioma:

Originated within the brain tissue.

A patient with neurological infection develops cerebral edema from syndrome of inappropriate antidiuretic hormone (SIADH). Which of the following is an important nursing action for this patient? a) Maintaining adequate hydration b) Administering prescribed antipyretics c) Restricting fluid intake and hydration d) Hyperoxygenation before and after tracheal suctioning

Restricting fluid intake and hydration Correct Explanation: Fluid restriction may be necessary if the patient develops cerebral edema and hypervolemia from SIADH. Antipyretics are administered to patients who develop hyperthermia. In addition, it is important to maintain adequate hydration in such patients. A patient with neurological infection should be given tracheal suctioning and hyperoxygenation only when the patient develops respiratory distress

The nurse is aware that, when assessing a patient for symptoms of a brain tumor, the symptom most frequently found is:

Simple to generalized seizures.

What basic information will the nurse caring for a patient recently diagnosed with multiple sclerosis (MS) provide to him? A) It is a degenerative disease of the nervous system. B) It usually occurs more frequently in men. C) It has an acute onset. D) It is caused by a bacterial infection.

Ans: A Feedback: Multiple sclerosis is a chronic, degenerative, progressive disease of the central nervous system (CNS) characterized by the occurrence of small patches of demyelination in the brain and spinal cord. The cause of MS is not known; it affects twice as many women as men.

A 42-year-old woman diagnosed with metastatic cancer has developed trigeminal neuralgia. She is taking carbamazepine (Tegretol) for pain relief. Which of the following applies to this medication? A) The medication should be taken on an empty stomach. B) Thee patient should be monitored for bone marrow depression.. C) Side effects include renal dysfunction. D) The medication should be taken in maximum dosage form to be effective.

Ans: B Feedback: The anticonvulsant agents carbamazepine (Tegretol) and phenytoin (Dilantin) relieve pain in most patients diagnosed with trigeminal neuralgia by reducing the transmission of impulses at certain nerve terminals. Carbamazepine is taken with meals and should be gradually increased until pain relief is obtained.

A 46-year-old client has been diagnosed with major depressive disorder. The client is seeing a nurse practitioner who is deciding on an appropriate treatment regimen. The nurse practitioner knows that which will be the most effective treatment for this client's depressive disorder?

A combination of psychotherapy and medication

A client with multiple sclerosis (MS) is experiencing bowel incontinence and is starting a bowel retraining program. Which strategy is not appropriate? a) limiting fluid intake to 1,000 mL/day b) setting a regular time for elimination c) using an elevated toilet seat d) eating a diet high in fiber

A) imiting fluid intake to 1,000 mL/day Explanation: Limiting fluid intake is likely to aggravate rather than relieve symptoms when a bowel retraining program is being implemented. Furthermore, water imbalance, as well as electrolyte imbalance, tends to aggravate the signs and symptoms of MS. A diet high in fiber helps keep bowel movements regular. Setting a regular time each day for elimination helps train the body to maintain a schedule. Using an elevated toilet seat facilitates transfer of the client from the wheelchair to the toilet or from a standing to a sitting position

A patient with metastatic cancer has developed trigeminal neuralgia and is taking carbamazepine (Tegretol) for pain relief. What principle applies to the administration of this medication? A) Tegretol is not known to have serious adverse effects. B) The patient should be monitored for bone marrow depression. C) Side effects of the medication include renal dysfunction. D) The medication should be first taken in the maximum dosage form to be effective.

B) The patient should be monitored for bone marrow depression.

Which of the following is a late symptom of spinal cord compression?

Paralysis pg. 2056

The physician has ordered a Tensilon test to rule out myasthenia gravis. The nurse knows that which of the following medications would be used to counteract the side effects of the Tensilon? A) Baclofen (Lioresal) B) Atropine (AtroPen) C) Epinephrine (Adrenalin) D) Narcan (Naloxone)

Ans: B Feedback: Atropine 0.4 mg controls the side effects of Tensilon, which include bradycardia, sweating, and cramping. Baclofen is a skeletal muscle relaxant used in the treatment of multiple sclerosis. Epinephrine is used in the treatment of anaphylaxis, cardiac arrest, and bronchospasm. Narcan is used to reverse the narcotic-induced respiratory depression.

You are the clinic nurse caring for a patient with a recent diagnosis of myasthenia gravis. The patient has begun treatment with pyridostigmine bromide (Mestinon). What change in status would most clearly suggest a therapeutic benefit of this medication? A) Increased muscle strength B) Decreased pain C) Improved GI function D) Improved cognition

A) Increased muscle strength

The nurse is caring for a client hospitalized with a severe exacerbation of myasthenia gravis. When administering medications to this client, what is a priority nursing action? a) Document medication given and dose. b) Assess client's reaction to new medication schedule. c) Administer medications at exact intervals ordered. d) Give client plenty of fluids with medications.

Administer medications at exact intervals ordered. Correct Explanation: He or she must administer medications at the exact intervals ordered to maintain therapeutic blood levels and prevent symptoms from returning. Assessing the client's reaction, documenting medication and dose, and giving the client plenty of fluids are not the priority nursing action for this client.

Medical management of BPH includes pharmacologic therapy. Which of the following medications would the nurse expect the health care provider to prescribe for this diagnosis?

Alpha-adrenergic blocker

The nurse is caring for a patient diagnosed with Parkinson's disease. The patient is having increasing problems with rising from the sitting to the standing position. What should the nurse suggest to the patient to use that will aid in getting from the sitting to the standing position as well as aid in improving bowel elimination? A) Use of a bedpan B) Use of a raised toilet seat C) Sitting quietly on the toilet every 2 hours D) Following the outlined bowel program

Ans: B Feedback: A raised toilet seat is useful, because the patient has difficulty in moving from a standing to a sitting position. A handicapped toilet is not high enough and will not aid in improving bowel elimination. Sitting quietly on the toilet every 2 hours will not aid in getting from the sitting to standing position; neither will following the outlined bowel program.

The nurse is caring for a patient who is scheduled for a cervical discectomy the following day. During health education, the patient should be made aware of what potential complications? A) Vertebral fracture B) Hematoma at the surgical site C) Scoliosis D) Renal trauma

Ans: B Feedback: Based on all the assessment data, the potential complications of diskectomy may include hematoma at the surgical site, resulting in cord compression and neurologic deficit and recurrent or persistent pain after surgery. Renal trauma and fractures are unlikely; scoliosis is a congenital malformation of the spine.

The nurse is caring for a boy who has muscular dystrophy. When planning assistance with the patient's ADLs, what goal should the nurse prioritize? A) Promoting the patient's recovery from the disease B) Maximizing the patient's level of function C) Ensuring the patient's adherence to treatment D) Fostering the family's participation in care

Ans: B Feedback: Priority for the care of the child with muscular dystrophy is the need to maximize the patient's level of function. Family participation is also important, but should be guided by this goal. Adherence is not a central goal, even though it is highly beneficial, and the disease is not curable.

A patient newly diagnosed with a cervical disk herniation is receiving health education from the clinic nurse. What conservative management measures should the nurse teach the patient to implement? A) Perform active ROM exercises three times daily. B) Sleep on a firm mattress. C) Apply cool compresses to the back of the neck daily. D) Wear the cervical collar for at least 2 hours at a time.

Ans: B Feedback: Proper positioning on a firm mattress and bed rest for 1 to 2 days may bring dramatic relief from pain. The patient may need to wear a cervical collar 24 hours a day during the acute phase of pain from a cervical disk herniation. Hot, moist compresses applied to the back of the neck will increase blood flow to the muscles and help relax the spastic muscles.

patient with amyotrophic lateral sclerosis (ALS) is being visited by the home health nurse who is creating a care plan. What nursing diagnosis is most likely for a patient with this condition? A) Chronic confusion B) Impaired urinary elimination C) Impaired verbal communication D) Bowel incontinence

Ans: C Feedback: Impaired communication is an appropriate nursing diagnosis; the voice in patients with ALS assumes a nasal sound and articulation becomes so disrupted that speech is unintelligible. Intellectual function is marginally impaired in patients with late ALS. Usually, the anal and bladder sphincters are intact because the spinal nerves that control muscles of the rectum and urinary bladder are not affected.

A patient with trigeminal neuralgia is taking Tegretol (carbamazepine) to alleviate pain associated with this disorder. It is important to teach the patient that which of the following side effects may occur from taking this medication? A) Skin discoloration B) Drowsiness C) Insomnia D) Tinnitus

Ans: B Feedback: Side effects of Tegretol include nausea, dizziness, drowsiness, and aplastic anemia. The patient must also be monitored for bone marrow depression during long-term therapy. Skin discoloration, insomnia, and tinnitus are not side effects of Tegretol.

A nurse is caring for a client admitted with a diagnosis of exacerbation of myasthenia gravis. Upon assessment of the client, the nurse notes the client has severely depressed respirations. The nurse would expect to identify which acid-base disturbance? a) Metabolic acidosis b) Metabolic alkalosis c) Respiratory alkalosis d) Respiratory acidosis

D) Respiratory acidosis Explanation: Respiratory acidosis is always from inadequate excretion of CO2 with inadequate ventilation, resulting in elevated plasma CO2 concentrations. Respiratory acidosis can occur in diseases that impair respiratory muscles such as myasthenia gravis.

When assessing an older adult's gastrointestinal system, the nurse would identify an increase in which of the following as normal? A) Feeling of fullness B) Gastric motility C) Calcium absorption D) Stomach emptying

Feeling of fullness

The nursing instructor gives their students an assignment of making a plan of care for a client with Huntington's disease. What would be important for the students to include in the teaching portion of the care plan?

How to facilitate tasks such as using both hands to hold a drinking glass

Which of the following is considered a central nervous system (CNS) disorder?

Multiple sclerosis

Which is the primary concern for a client with panic-level anxiety?

Safety

Which patient will the nurse assess for degenerative neurologic symptoms?

The patient with Huntington disease

Nursing students are reviewing information about Parkinson's disease in preparation for class the next day. The students demonstrate understanding of the material when they identify which of the following as a cardinal sign of this disorder? Select all that apply.

Tremor, Ridigity, Bradykinesia,Postural instability

A 42-year-old client has been prescribed a monoamine oxidase inhibitor (MAOI). The client should be informed to avoid foods containing what?

Tyramine

Based on the nurse's knowledge of the probable first indicator of prostate cancer, which of the following questions should be included in the history and physical examination?

"Do you have any perineal discomfort?"

A 17-year-old adolescent with a history of muscular dystrophy is admitted with aspiration pneumonia. The nurse asks the parents if the client has an advance directive. Which response by the parents leads the nurse to believe that the parents don't understand the severity of the client's medical condition?

"He is only 17. He doesn't need an advance directive."

Which question in the assessment of a client with anxiety is most clinically appropriate?

"How do you feel about everything that is happening in your life right now?"

A client comes to the clinic reporting urinary symptoms. Which statement would most likely alert the nurse to suspect benign prostatic hyperplasia (BPH)?

"I've had trouble getting started when I urinate, often straining to do so."

An older adult develops sudden onset of confusion and is hospitalized. The family expresses concern that their loved one is developing Alzheimer disease. What response by the nurse is most appropriate?

"Several possible underlying factors could be causing the confusion. Alzheimer's usually does not present with sudden confusion."

A client with Alzheimer's disease is prescribed donepezil hydrochloride. When teaching the client and family about this drug, which of the following would the nurse include?

"The drug helps to control the symptoms of the disease."

A patient with Alzheimer's disease is prescribed donepezil (Aricept). When teaching the patient and family about this drug, which of the following would the nurse include? "This drug will help to stop the disease from getting worse." "The drug helps to control the symptoms of the disease." "Once it becomes effective, you can stop the drug." "He'll need to take this drug for the rest of his life."

"The drug helps to control the symptoms of the disease."

A client is prescribed dutasteride (Avodart) as part of treatment for benign prostatic hyperplasia (BPH). When teaching the client about this drug, the nurse integrates knowledge of the drug's action. Which statement best reflects this?

"The drug interferes with testosterone, which causes the prostate to shrink in size."

A client with a malignant glioma is scheduled for surgery. The client demonstrates a need for additional teaching about the surgery when he states which of the following?

"The surgeon will be able to remove all of the tumor." pg. 2053

The parents of a patient intubated due to the progression of Guillain-Barré syndrome ask if their child will die. What is the best response by the nurse?

"There are no guarantees, but a large portion of people with Guillain-Barré syndrome survive."

The nurse is caring for a patient in the neurologic ICU who sustained head trauma in a physical altercation. What would the nurse know is an optimal range of ICP for this patient? a) 20 to 30 mm Hg b) 8 to 15 mm Hg c) 25 to 40 mm Hg d) 0 to 10 mm Hg

0 to 10 mm Hg Explanation: ICP is usually measured in the lateral ventricles, with the normal pressure being 0 to 10 mm Hg, and 15 mm Hg being the upper limit of normal (Hickey, 2009).

A nurse assesses the patient's LOC using the Glasgow Coma Scale. What score indicates severe impairment of neurologic function? a) 12 b) 6 c) 3 d) 9

3 Correct Explanation: LOC, a sensitive indicator of neurologic function, is assessed based on the criteria in the Glasgow Coma Scale: eye opening, verbal response, and motor response (Barlow, 2012). The patient's responses are rated on a scale from 3 to 15. A score of 3 indicates severe impairment of neurologic function, brain death, or pharmacologic inhibition of the neurologic response. A score of 15 indicates that the patient is fully responsive (see Chapter 68).

A patient is admitted to the hospital with an ICP reading of 20 mm Hg and a mean arterial pressure of 90 mm Hg. What would the nurse calculate the CPP to be? a) 70 mm Hg b) 60 mm Hg c) 50 mm Hg d) 80 mm Hg

70 mm Hg Explanation: Changes in ICP are closely linked with cerebral perfusion pressure (CPP). The CPP is calculated by subtracting the ICP from the mean arterial pressure (MAP). For example, if the MAP is 100 mm Hg and the ICP is 15 mm Hg, then the CPP is 85 mm Hg. The normal CPP is 70 to 100 mm Hg (Hickey, 2009).

A nurse knows that a patient exhibiting seizurelike movements localized to one side of the body most likely has what type of tumor?

A motor cortex tumor A tumor in the motor cortex of the frontal lobe produces hemiparesis and partial seizures on the opposite side of the body or generalized seizures. A frontal lobe tumor may also produce changes in emotional state and behavior, as well as an apathetic mental attitude. A cerebellar tumor causes dizziness; an ataxic or staggering gait with a tendency to fall toward the side of the lesion; marked muscle incoordination; and nystagmus (involuntary rhythmic eye movements), usually in the horizontal direction. An occipital lobe tumor produces visual manifestations: contralateral homonymous hemianopsia (visual loss in half of the visual field on the opposite side of the tumor) and visual hallucinations.

A patient diagnosed with Bell's palsy is being cared for on an outpatient basis. During health education, the nurse should promote which of the following actions? A) Applying a protective eye shield at night B) Chewing on the affected side to prevent unilateral neglect C) Avoiding the use of analgesics whenever possible D) Avoiding brushing the teeth

A) Applying a protective eye shield at night

Which intervention should the nurse suggest to help a client with multiple sclerosis avoid episodes of urinary incontinence? a) Establish a regular voiding schedule. b) Insert an indwelling urinary catheter. c) Limit fluid intake to 1,000 mL/day. d) Administer prophylactic antibiotics, as prescribed.

A) Establish a regular voiding schedule. Explanation: Maintaining a regular voiding pattern is the most appropriate measure to help the client avoid urinary incontinence. Fluid intake is not related to incontinence. Incontinence is related to the strength of the detrusor and urethral sphincter muscles. Inserting an indwelling catheter would be a treatment of last resort because of the increased risk of infection. If catheterization is required, intermittent self-catheterization is preferred because of its lower risk of infection. Antibiotics do not influence urinary incontinence

A patient with diabetes presents to the clinic and is diagnosed with a mononeuropathy. This patient's nursing care should involve which of the following? A) Protection of the affected limb from injury B) Passive and active ROM exercises for the affected limb C) Education about improvements to glycemic control D) Interventions to prevent contractures

A) Protection of the affected limb from injury

Which is an initial sign of Parkinson's disease? a) tremor b) bradykinesia c) rigidity d) akinesia

A) tremor Explanation: The first sign of Parkinson's disease is usually tremors. The client commonly is the first to notice this sign because the tremors may be minimal at first. Rigidity is the second sign, and bradykinesia is the third sign. Akinesia is a later stage of bradykinesia.

Which of the following is a disease in which there is a loss of motor neurons in the anterior horns of the spinal cord and motor nuclei of the lower brain stem?

Amyotrophic lateral sclerosis (ALS)

A nurse is assessing a patient with an acoustic neuroma who has been recently admitted to an oncology unit. What symptoms is the nurse likely to find during the initial assessment? A) Loss of hearing, tinnitus, and vertigo B) Loss of vision, change in mental status, and hyperthermia C) Loss of hearing, increased sodium retention, and hypertension D) Loss of vision, headache, and tachycardia

Ans: A Feedback: An acoustic neuroma is a tumor of the eighth cranial nerve, the cranial nerve most responsible for hearing and balance. The patient with an acoustic neuroma usually experiences loss of hearing, tinnitus, and episodes of vertigo and staggering gait. Acoustic neuromas do not cause loss of vision, increased sodium retention, or tachycardia.

Which of the following is a clinical manifestation associated with Guillain-Barré syndrome? A) Vertigo B) Ptosis of the eyelid C) Diminished taste for food D) Vocal paralysis

Ans: D Feedback: Guillain-Barré syndrome is a disorder of the vagus nerve. Clinical manifestations include vocal paralysis, dysphagia, and voice changes (temporary or permanent hoarseness).

Which of the following nursing interventions is appropriate for a patient with double vision in the right eye due to MS?

Apply an eye patch to the right eye.

A client reports to the nurse that her elderly mother has become increasingly angry and responds inappropriately to conversations within the past few months. She notes that her mother does not respond when the mother's back is turned. The best intervention of the nurse is to

Ask if the mother could come in for a hearing evaluation

A client reports to the nurse that over the past few months the elderly mother has become increasingly angry, responds inappropriately to conversations, and does not respond to calls if her back is turned away. What is the nurse's best response?

Ask if the mother could come in for a hearing evaluation.

A patient with herpes simplex virus encephalitis (HSV) has been admitted to the ICU. What medication would the nurse expect the physician to order for the treatment of this disease process? A) Cyclosporine (Neoral) B) Acyclovir (Zovirax) C) Cyclobenzaprine (Flexeril) D) Ampicillin (Prinicpen)

B) Acyclovir (Zovirax)

A patient with MS has developed dysphagia as a result of cranial nerve dysfunction. What nursing action should the nurse consequently perform? A) Arrange for the patient to receive a low residue diet. B) Position the patient upright during feeding. C) Suction the patient following each meal. D) Withhold liquids until the patient has finished eating.

B) Position the patient upright during feeding.

The nurse is caring for a patient with MS who is having spasticity in the lower extremities that decreases physical mobility. What interventions can the nurse provide to assist with relieving the spasms? (Select all that apply.) A. have the patient take a hot tub bath to allow muscle relaxation B. demonstrate daily muscle stretching exercises C. apply warm compresses to the affected areas D. allow the patient adequate time to perform exercises E. assist with a rigorous exercise program to prevent contractures

B, C, D

A patient diagnosed with a tumor in the cerebellar region would expect to have changes in which of the following?

Balance and coordination

During the Tensilon test to determine if the patient has myasthenia gravia, the patient complains of cramping and becomes diaphoretic. Vital signs are BP 130/78, HR 42 and respiration 18. What intervention should the nurse prepare to do? A. place the patient in the supine position B. administer diphenhyramine (Benadryl) for the allergic reaction C. administer atropine to control the side effects of the edrophonium D. call the rapid response team because the patient is preparing to arrest

C

The nurse is administering the IV antiviral medication ganciclovir (Cytovene) to the patient with HSV-1 encephalitis. What is the best way for the nurse to administer the medication to avoid crystallization of the medication in the urine? A. Administer the medication rapidly over 15 minutes with 100 mL of normal saline. B. Dilute the medicine in 500 mL of lactated Ringer's solution C. Administer via slow IV over 1 hour D. Administer in a drip over 4 hours

C

The nurse is caring for the patient in the emergency department with an onset of pain related to trigeminal neuralgia. What subjective data stated by the patient does the nurse determine triggered the paroxysms of pain? A. "I was sitting at home watching television." B. "I was putting my shoes on." C. "I was brushing my teeth." D. "I was taking a bath."

C

After teaching an older adult about measures to relieve constipation, which statement by the client indicates a need for additional teaching? A) "I should consume more foods with fat." B) "I should drink more water." C) "I should use a laxative every other day." D) "I should exercise every other day."

C) "I should use a laxative every other day."

A 33-year-old patient presents at the clinic with complaints of weakness, incoordination, dizziness, and loss of balance. The patient is hospitalized and diagnosed with MS. What sign or symptom, revealed during the initial assessment, is typical of MS? A) Diplopia, history of increased fatigue, and decreased or absent deep tendon reflexes B) Flexor spasm, clonus, and negative Babinski's reflex C) Blurred vision, intention tremor, and urinary hesitancy D) Hyperactive abdominal reflexes and history of unsteady gait and episodic paresthesia in both legs

C) Blurred vision, intention tremor, and urinary hesitancy

A patient is admitted through the ED with suspected St. Louis encephalitis. The unique clinical feature of St. Louis encephalitis will make what nursing action a priority? A) Serial assessments of hemoglobin levels B) Blood glucose monitoring C) Close monitoring of fluid balance D) Assessment of pain along dermatomes

C) Close monitoring of fluid balance

A patient with Guillain-Barré syndrome has experienced a sharp decline in vital capacity. What is the nurse's most appropriate action? A) Administer bronchodilators as ordered. B) Remind the patient of the importance of deep breathing and coughing exercises. C) Prepare to assist with intubation. D) Administer supplementary oxygen by nasal cannula.

C) Prepare to assist with intubation.

The critical care nurse is admitting a patient in myasthenic crisis to the ICU. The nurse should prioritize what nursing action in the immediate care of this patient? A) Suctioning secretions B) Facilitating ABG analysis C) Providing ventilatory assistance D) Administering tube feedings

C) Providing ventilatory assistance

A nurse is teaching a nutrition class to a group of elderly clients at a retirement center. The nurse tells the clients that as a result of aging, they will need to decrease... A) Activity level B) Calories C) Proteins D) Complex carbohydrates

Calories Chapter 11: Health Care of the Older Adult - Page 199

Which of the following is a rapid, jerky, involuntary, and purposeless movement of the extremities or facial muscles?

Chorea

The nurse identifies a nursing diagnosis of self-care deficit, bathing related to motor impairment and decreased cognitive function for a client with cerebral metastasis. Which outcome would the nurse most likely identify on this client's plan of care?

Client participates in daily hygiene activities with assitive devices.

The nurse walks into the client's room and finds the client sobbing uncontrollably. When the nurse asks what the problem is, the client responds, "I am so scared. I have never known anyone who goes into a hospital and comes out alive." On this client's care plan the nurse notes a nursing diagnosis of ineffective coping related to stress. What is the best outcome to be expected for this client?

Client will adapt relaxation techniques to reduce stress.

An adult client was admitted to the psychiatric mental health unit following a suicide attempt. The client has responded well to treatment, so discharge is being considered. In anticipation of the client's discharge, the nurse should:

Collaborate with the family to make sure the client's home environment is safe.

A client is admitted to an acute care facility after an episode of status epilepticus. After the client is stabilized, which factor is most beneficial in determining the potential cause of the episode? a) The type of anticonvulsant prescribed to manage the epileptic condition b) Recent weight gain and loss c) Compliance with the prescribed medication regimen d) Recent stress level

Compliance with the prescribed medication regimen Correct Explanation: The most common cause of status epilepticus is sudden withdraw of anticonvulsant therapy. The type of medication prescribed, the client's stress level, and weight change don't contribute to this condition.

The nurse is volunteering for a Red Cross blood drive and is taking the history or potential donors. Which volunteer would the nurse know will not be allowed to donate blood? A. a donor with a history of hypertension with a blood pressure of 140/90 mm Hg B. a donor who is taking medication for benign prostatic hyperplasia C. a donor who moved to the United States from Canada D. a donor who was in college in England for 1 year

D

The nurse caring for a patient in ICU diagnosed with Guillain-Barré syndrome should prioritize monitoring for what potential complication? A) Impaired skin integrity B) Cognitive deficits C) Hemorrhage D) Autonomic dysfunction

D) Autonomic dysfunction

A patient with suspected Creutzfeldt-Jakob disease (CJD) is being admitted to the unit. The nurse would expect what diagnostic test to be ordered for this patient? A) Cerebral angiography B) ABG analysis C) CT D) EEG

D) EEG

The nurse is creating a plan of care for a patient who has a recent diagnosis of MS. Which of the following should the nurse include in the patient's care plan? A) Encourage patient to void every hour. B) Order a low-residue diet. C) Provide total assistance with all ADLs. D) Instruct the patient on daily muscle stretching.

D) Instruct the patient on daily muscle stretching.

A 73-year-old man comes to the clinic complaining of weakness and loss of sensation in his feet and legs. Assessment of the patient shows decreased reflexes bilaterally. Why would it be a challenge to diagnose a peripheral neuropathy in this patient? A) Older adults are often vague historians. B) The elderly have fewer peripheral nerves than younger adults. C) Many older adults are hesitant to admit that their body is changing. D) Many symptoms can be the result of normal aging process.

D) Many symptoms can be the result of normal aging process.

A patient presents at the clinic complaining of pain and weakness in her hands. On assessment, the nurse notes diminished reflexes in the upper extremities bilaterally and bilateral loss of sensation. The nurse knows that these findings are indicative of what? A) Guillain-Barré syndrome B) Myasthenia gravis C) Trigeminal neuralgia D) Peripheral nerve disorder

D) Peripheral nerve disorder

The nurse is caring for a client hospitalized after a motor vehicle accident. The client has a comorbidity of Parkinson's disease. Why should the nurse closely monitor the condition and the drug regimen of a client with Parkinson's disease?

Drugs administered may cause a wide variety of adverse effects.

A client with Parkinson's disease has been receiving levodopa as treatment for the past 7 years. The client comes to the facility for an evaluation and the nurse observes facial grimacing, head bobbing, and smacking movements. The nurse interprets these findings as which of the following?

Dyskinesia

The nurse is performing an assessment for a patient in the clinic with Parkinson's disease. The nurse determines that the patient's voice has changed since the last visit and is now more difficult to understand. How should the nurse document this finding?

Dysphonia Dysphonia (voice impairment or altered voice production) may occur as a result of weakness and incoordination of the muscles responsible for speech.

A male client who is admitted with the diagnosis of urinary calculi complains of excruciating pain. The pain is suspected to be caused by increased pressure in the renal pelvis. Which measure would be most appropriate to provide pain relief?

Encourage frequent ambulation.

Bell's palsy is a disorder of which cranial nerve?

Facial (VII)

While performing an initial nursing assessment on a client admitted with suspected tic douloureux (trigeminal neuralgia), for which of the following would the nurse expect to observe?

Facial pain in the areas of the fifth cranial nerve

A nurse providing community education for parents regarding adolescent suicide should include in the teaching session that the most frequent cause or motive for suicide in this age group is what?

Feelings of alienation or isolation

A 48-year-old man recently diagnosed with benign prostatic hyperplasia (BPH) reports consuming <i>Serenoa repens</i> (saw palmetto berry). The nurse needs to intervene if the physician orders which treatment?

Finasteride

The client with benign prostatic hyperplasia (BPH) is considering use of medication in the management of symptoms. Which of the following drugs reduces the size of the prostate without lowering circulating levels of testosterone?

Finasteride (Proscar)

A client has no expression when conversing with the nurse. This would be documented as which type of affect?

Flat

A client with Guillain-Barre syndrome cannot swallow and has a paralytic ileus; the nurse is administering parenteral nutrition intraveneously. The nurse is careful to assess which of the following related to intake of nutrients?

Gag reflex and bowel sounds

A client is admitted reporting low back pain. How will the nurse best determine if the pain is related to a herniated lumbar disc?

Have the client lie on the back and lift the leg, keeping it straight. pg. 2074

The nurse is assessing a client with meningitis. Which of the following signs would the nurse expect to observe?

Headache and nuchal rigidity

A patient has recently been diagnosed with an acoustic neuroma. The nurse helps the patient understand that:

Hearing loss usually occurs.

What is the leading cause of death among the elderly? A) Heart disease B) Cancer C) Pneumonia D) Cerebrovascular disease

Heart disease

A nurse is educating a group of middle-aged adults on aging. What information should the nurse include in the teaching? A decline in sexual activity is a normal occurrence as you age. Most older adults reside in a long-term care facility. How old you feel will be determined by your physical and cognitive abilities. As an older adult, you will not be able to learn new skills or knowledge.

How old you feel will be determined by your physical and cognitive abilities.

The nurse is caring for a patient postoperatively after intracranial surgery for the treatment of a subdural hematoma. The nurse observes an increase in the patient's blood pressure from the baseline and a decrease in the heart rate from 86 to 54. The patient has crackles in the bases of the lungs. What does the nurse suspect is occurring? a) Increased ICP b) Exacerbation of uncontrolled hypertension c) Increase in cerebral perfusion pressure d) Infection

Increased ICP Explanation: Increased ICP and bleeding are life threatening to the patient who has undergone intracranial surgery. An increase in blood pressure and decrease in pulse with respiratory failure may indicate increased ICP.

When assessing a client with benign prostatic hyperplasia, which of the following would the nurse expect the client to report as the initial complaint?

Increased effort to void

The nurse is planning a presentation to a group of nursing students on the topic of anxiety disorders. Which would the nurse include when describing panic disorder?

Individuals may believe they are having a heart attack when a panic attack occurs.

A patient has been brought to the ED with altered LOC, high fever, and a purpura rash on the lower extremities. The family states the patient was complaining of neck stiffness earlier in the day. What action should the nurse do first?

Initiate isolation precautions.

Which of the following is the only known risk factor for brain tumors?

Ionizing radiation

The nurse is providing client teaching to a client with early stage Alzheimer disease (AD) and her family. The client has been prescribed donepezil hydrochloride. What should the nurse explain to the client and family about this drug?

It slows the progression of AD.

Environmental factors may be associated with suicidal behavior. Which is an environmental factor?

Job loss

The nurse is caring for a patient with severe pain related to ureteral colic. What medication can the nurse administer with a physician's order that will inhibit the synthesis of prostaglandin E, thereby reducing swelling and facilitating passage of the stone?

Ketoralac (Toradol)

What does the nurse recognize as the earliest sign of serious impairment of brain circulation related to increasing ICP? a) Lethargy and stupor b) Hypertension c) Bradycardia d) A bounding pulse

Lethargy and stupor Correct Explanation: As ICP increases, the patient becomes stuporous, reacting only to loud or painful stimuli. At this stage, serious impairment of brain circulation is probably taking place, and immediate intervention is required.

Which of the following medications is the most effect agent in the treatment of Parkinson's disease (PD)?

Levodopa (Larodopa)

A client with weakness and tingling in both legs is admitted to the medical-surgical unit with a tentative diagnosis of Guillain-Barré syndrome. On admission, which assessment is most important for this client?

Lung auscultation and measurement of vital capacity and tidal volume

Which diagnostic is most commonly used for spinal cord compression?

Magnetic resonance imaging (MRI) pg. 2056

Which is a chronic, degenerative, progressive disease of the central nervous system characterized by the occurrence of demyelination in the brain and spinal cord?

Multiple sclerosis (MS)

When educating a patient about the use of antiseizure medication, what should the nurse inform the patient is a result of long-term use of the medication in women? a) Obesity b) Osteoarthritis c) Anemia d) Osteoporosis

Osteoporosis Correct Explanation: Because of bone loss associated with the long-term use of antiseizure medications, patients receiving antiseizure agents should be assessed for low bone mass and osteoporosis. They should be instructed about strategies to reduce their risks of osteoporosis (AANN, 2009).

A client seeks care for lower back pain of 2 weeks' duration. Which assessment finding suggests a herniated intervertebral disk?

Pain radiating down the posterior thigh

Which of the following diseases is associated with decreased levels of dopamine due to destruction of pigmented neuronal cells in the substantia nigra in the basal ganglia of the brain?

Parkinson's disease

The nurse is assessing a newly admitted client with a diagnosis of meningitis. On assessment, the nurse expects to find which of the following?

Positive Kernig's sign

The nurse is caring for a patient with Huntington disease. What intervention is a priority for safe care?

Protecting the patient from falls

What is a nurse's role in providing home care for a client with Alzheimer disease?

Provide emotional and physical support.

The nurse is caring for a patient with Huntington's disease in the long-term care facility. What does the nurse recognize as the most prominent symptom of the disease that the patient exhibits?

Rapid, jerky, involuntary movements

A client with an intravenous infusion is rubbing his arm. The nurse assesses the site and decides to discontinue the current infusion because of concern that the client has developed phlebitis. Which of the following clinical manifestations would the nurse assess with phlebitis? Select all that apply. Cool area around the insertion site Reddened area along the path of the vein Tender area around the insertion site Ecchymosis at the insertion site Rapid, shallow respirations

Reddened area along the path of the vein Tender area around the insertion site

Which is a factor that contributes to urinary incontinence in older female adults?

Relaxed perineal muscle

A client with Alzheimer's disease is admitted for hip surgery after falling and fracturing the right hip. The client's spouse tells the nurse about feeling guilty for letting the accident happen and reports not sleeping well lately because the spouse has been getting up at night and doing odd things. Which nursing diagnosis is most appropriate for the client's spouse?

Risk for caregiver role strain related to increased client care needs

Which of the following herbal remedies is used to treat symptoms of benign prostatic hypertrophy (BPH)?

Saw palmetto

According to the biochemical theory of mood disorders, a client with a diagnosis of depression is likely to have alterations in the levels and function of which neurotransmitters?

Serotonin, norepinephrine, and dopamine

A 30-year-old client who has been unemployed secondary to the client's anxiety disorder states that the client would like to have a job where the client is alone and no one needs to evaluate the client's work. The nurse interprets these comments as an indicator of what?

Social anxiety disorder

Which of the following positions should be utilized after supratentorial intracranial surgery? a) Body and head aligned b) Bed rest with a firm mattress and bed board c) Sitting position d) Supine position with head slightly elevated

Supine position with head slightly elevated Correct Explanation: After surgery, the nurse should place the patient in either a supine position with the head slightly elevated or a side-lying position on the unaffected side. Bed rest with a firm mattress and bed board is used for patients with a lumbar herniated disk. Sitting position and body and head aligned are the correct positions to place the patient after the surgery.

The nurse understands that a certain level of anxiety is required in a client for effective learning. Which anxiety-related symptom indicates the client may be able to learn effectively?

The client has heightened awareness.

Which client goal, established by the nurse, is most important as the nurse plans care for a seizure client in the home setting?

The client will remain free of injury if a seizure does occur.

The nurse is conducting a mental status assessment with a client who has major depressive disorder. The client tells the nurse, "I really feel like I can't take it anymore." What additional information should the nurse seek out in order to determine if an emergency alert is warranted?

The client's history of previous suicide attempts

A patient with Parkinson's disease is experiencing an on-off syndrome. What does the nurse recognize that the patient's clinical symptoms will be?

The patient will have periods of near immobility, followed by a sudden return of effectiveness of the medication. pg. 2064

A client comes to the emergency department complaining of a sudden onset of sharp, severe flank pain. During the physical examination, the client indicates that the pain, which comes in waves, travels to the suprapubic region. He states, "I can even feel the pain at the tip of my penis." Which of the following would the nurse suspect?

Urinary calculi

The following information is documented on the assessment form for an older adult: Kyphosis Dry mucous membranes Decreased respiratory excursion Urinary incontinence The nurse is reviewing the information and reports which of these findings to the physician?

Urinary incontinence

A patient had a small pituitary adenoma removed by the transsphenoidal approach and has developed diabetes insipidus. What pharmacologic therapy will the nurse be administering to this patient to control symptoms? a) Vasopressin b) Furosemide (Lasix) c) Mannitol d) Phenobarbital

Vasopressin Correct Explanation: Manipulation of the posterior pituitary gland during surgery may produce transient diabetes insipidus of several days' duration (Hickey, 2009). It is treated with vasopressin but occasionally persists.

A nurse is caring for a client with a history of severe migraines. The client has a medical history that includes asthma, gastroesophageal reflux disease, and three pregnancies. Which medication does the nurse anticipate the physician will order for the client's migraines? a) Amiodarone (Cordarone) b) Verapamil (Calan) c) Metoprolol (Lopressor) d) Captopril (Coreg)

Verapamil (Calan) Explanation: Calcium channel blockers, such as verapamil, and beta-adrenergic blockers, such as metoprolol, are commonly used to treat migraines because they help control cerebral blood vessel dilation. Calcium channels blockers, however, are ordered for clients who may not be able to tolerate beta-adrenergic blockers, such as those with asthma. Amiodarone and captopril aren't used to treat migraines.

A client reports experiencing nocturia. The nurse obtains a thorough history including current signs and symptoms. What would the client likely include when describing the symptoms?

decrease in the force of urinary stream

A nurse is caring for an elderly adult client admitted to the hospital from a nursing home because of a change in behavior. The client has a diagnosis of Alzheimer's disease and has started to experience episodes of incontinence. The hospital staff is having difficulty with toileting because the client wanders around the unit all day. To assist with elimination, a nurse should: A) have the client wear two briefs at a time to ensure absorption of incontinent urine. B) ask the physician to order sedation to allow the client to rest. C) incorporate the client's toileting schedule into the pattern of his wandering. D) ask the physician to order restraints to prevent wandering.

incorporate the client's toileting schedule into the pattern of his wandering.

A nurse is working on a surgical floor. The nurse must logroll a client following a:

laminectomy

Bone density testing in patients with post-polio syndrome has demonstrated

low bone mass and osteoporosis.

The primary North American vector transmitting arthropod-borne virus encephalitis is the

mosquito

A nurse is providing postprocedure care for a client who underwent percutaneous lithotripsy. In this procedure, an ultrasonic probe inserted through a nephrostomy tube into the renal pelvis generates ultra-high-frequency sound waves to shatter renal calculi. The nurse should instruct the client to:

notify the physician about cloudy or foul-smelling urine.

Nursing students are reviewing information about Parkinson's disease in preparation for class the next day. The students demonstrate understanding of the material when they identify which of the following as a cardinal sign of this disorder? Select all that apply.

• Bradykinesia • Tremor • Rigidity • Postural instability

A client reports to the nurse that her grandmother with Alzheimer's disease recently moved in with her and her two school-aged children. The client states the grandmother becomes agitated and starts yelling and crying frequently. The woman asks, "What can I do?" The nurse first responds: A) "What precipitates the outbursts?" B) "You need to remain calm during the outbursts." C) "Play quiet music that your grandmother may like." D) "Start rubbing her shoulders and her back."

"What precipitates the outbursts?" Chapter 11: Health Care of the Older Adult - Page 213

A client who has just been diagnosed with mixed muscular dystrophy asks the nurse about the usual course of this disease. How should the nurse respond?

"You may experience progressive deterioration in all voluntary muscles."

The nurse is providing teaching to a client with depression. Which statement by the client would indicate that the education has been effective?

"I didn't realize so many factors could cause depression."

An older adult has encouraged her husband to visit their primary care provider, stating that she is concerned that he may have Parkinson's disease. Which of the wife's descriptions of her husband's health and function is most suggestive of Parkinson's disease? A) "Lately he seems to move far more slowly than he ever has in the past." B) "He often complains that his joints are terribly stiff when he wakes up in the morning." C) "He's forgotten the names of some people that we've known for years." D) "He's losing weight even though he has a ravenous appetite."

Ans: A Feedback: Parkinson's disease is characterized by bradykinesia. It does not manifest as memory loss, increased appetite, or joint stiffness.

A 37-year-old teacher is hospitalized with complaints of weakness, incoordination, dizziness, and loss of balance. The diagnosis is multiple sclerosis (MS). Which of the following signs and symptoms, revealed during the history and physical assessment, is typical of MS? A) Diplopia, history of increased fatigue, and decreased or absent deep tendon reflexes B) Flexor spasm, clonus, and negative Babinski's reflex C) Blurred vision, intention tremor, and urinary hesitancy D) Hyperactive abdominal reflexes and history of unsteady gait and episodic paresthesia in both legs

Ans: C Feedback: Optic neuritis, leading to blurred vision, is a common early sign of MS, as is intention tremor (tremor when performing an activity). Nerve damage can cause urinary hesitancy. In MS, deep tendon reflexes are increased or hyperactive. A positive Babinski's reflex is found in MS. Abdominal reflexes are absent with MS.

A 69-year-old patient is brought to the ED by ambulance because a family member found him lying on the floor disoriented and lethargic. The physician suspects bacterial meningitis and admits the patient to the ICU. The nurse knows that risk factors for an unfavorable outcome include what? Select all that apply. A) Blood pressure greater than 140/90 mm Hg B) Heart rate greater than 120 bpm C) Older age D) Low Glasgow Coma Scale E) Lack of previous immunizations

B) Heart rate greater than 120 bpm C) Older age D) Low Glasgow Coma Scale

The nurse is caring for a client admitted with a diagnosis of septic meningitis. The nurse is aware that this infection is caused by which of the following?

Bacteria

Which of the following is an anticholinergic agent used to control tremor and rigidity in Parkinson's disease?

Benztropine Mesylate (Cogentin)

The nurse is assessing the genitourinary status of an older adult female patient who is experiencing stress incontinence. What finding is a common gerontologic finding for this population? A) Renal filtration rate increases B) Urine is more dilute in the older population. C) Bladder capacity decreases with advanced age. D) All patients develop urinary tract infections.

Bladder capacity decreases with advanced age.

A client with a cerebellar brain tumor is admitted to an acute care facility. The nurse formulates a nursing diagnosis of Risk for injury. Which "related-to" phrase should the nurse add to complete the nursing diagnosis statement?

Related to impaired balance pg. 2055

A nurse is planning discharge teaching for an older adult client with mild short-term memory loss. The discharge teaching will include how to perform basic wound care for the venous ulcer on the client's lower leg. When planning the necessary health education for this client, the nurse should:

keep teaching periods short.

A nurse is reviewing the medications of a client who lives alone and reports having difficulty remembering when to take them. To aid in medication compliance, which of the following measures would the nurse employ? Select all answers that apply. A) Suggest that the client use a multiple-dose medication dispenser. B) Encourage the client to use containers with safety lids. C) Recommend to the client to use one pharmacy for all prescriptions. D) Write down the medication schedule for the client.

A) Write down the medication schedule for the client. C) Suggest that the client use a multiple-dose medication dispenser. D) Recommend to the client to use one pharmacy for all prescriptions.

Myasthenia gravis occurs when antibodies attack which receptor sites?

Acetylcholine

Which of the following nursing interventions would be included in the care plan for a patient admitted with MS? A) Encourage the patient to void 1 hour after drinking. B) Order a low-residue diet. C) Provide total assistance as needed with all activities of daily living. D) Instruct the patient on daily muscle stretching.

Ans: D Feedback: A patient diagnosed with MS should be encouraged to increase the fiber in his or her diet and void 30 minutes after drinking to help train the bladder. The patient should participate in daily muscle stretching to help alleviate and relax muscle spasms.

A hospitalized client states that the client is having difficulty resting. Which intervention would help promote rest?

Assisting the client with deep-breathing exercises

A patient diagnosed with Bell's palsy is having decreased sensitivity to touch of the involved nerve. What should the nurse recommend to prevent atrophy of the muscles? A) Blowing up balloons B) Deliberately frowning C) Smiling repeatedly D) Whistling

D) Whistling

The primary nursing goal for a client with myasthenia gravis is to: a) provide psychological support and reassurance. b) promote comfort and relieve pain. c) ensure a safe environment. d) maintain respiratory function.

D) maintain respiratory function. Explanation: In myasthenia gravis, major respiratory complications can result from weakness in the muscles of breathing and swallowing. The client is at risk for aspiration, respiratory infection, and respiratory failure. Providing a safe environment and emotional support are secondary goals. Pain is not commonly associated as a problem of myasthenia gravis.

Which goal is the most realistic for a client diagnosed with Parkinson's disease? a) to cure the disease b) to begin preparations for terminal care c) to stop progression of the disease d) to maintain optimal body function

D) to maintain optimal body function Explanation: Helping the client function at his or her best is most appropriate and realistic. There is no known cure for Parkinson's disease. Parkinson's disease progresses in severity, and there is no known way to stop its progression. However, many clients live for years with the disease: and it would not be appropriate to start planning terminal care at this time

A client was running along an ocean pier, tripped on an elevated area of the decking, and struck his head on the pier railing. According to his friends, "He was unconscious briefly and then became alert and behaved as though nothing had happened." Shortly afterward, he began complaining of a headache and asked to be taken to the emergency department. If the client's intracranial pressure (ICP) is increasing, the nurse should expect to observe which sign first? a) Irregular breathing pattern b) Pupillary asymmetry c) Involuntary posturing d) Declining level of consciousness (LOC)

Declining level of consciousness (LOC) Correct Explanation: With a brain injury such as an epidural hematoma (a likely diagnosis, based on this client's symptoms), the initial sign of increasing ICP is a change in LOC. As neurologic deterioration progresses, manifestations involving pupillary symmetry, breathing patterns, and posturing will occur.

Which condition is characterized by a decline in intellectual functioning? Delirium Delusion Dementia Depression

Dementia

The nurse is teaching about preventing pneumonia and influenza to a group of clients in a senior citizens' wellness class. The nurse includes which of the following topics in the class? Select all options that apply. Following a high-calcium diet Ensuring appropriate fluid intake Participating in regular exercise Avoiding all sun exposure Avoiding environmental smoke

Ensuring appropriate fluid intake Participating in regular exercise Avoiding environmental smoke

A health care provider needs help in identifying the precise location of a brain tumor. To measure brain activity, as well as to determine structure, the nurse expects the health care provider to order which of the following tests?

Positron-emission tomography (PET)

A 27-year-old woman has a 4-month-old baby. For the past 3 months, the client has been experiencing intense sadness, anxiety, and hopelessness. After having thoughts of killing her baby, she decided to seek help. What is the likely the cause of this client's experience?

Postpartum depression

Which is an age-related change associated with the nervous system? Cerebral hypertrophy Increased nerve impulse conduction Increased cerebral function Postural hypotension

Postural hypotension

Which refers to the decrease in lens flexibility that occurs with age, resulting in the near point of focus getting farther away?

Presbyopia

Which refers to the decrease in lens flexibility that occurs with age, resulting in the near point of focus getting farther away? Cataract Presbycusis Glaucoma Presbyopia

Presbyopia

The nurse is attempting to take vital signs of an older adult hospitalized following knee surgery. The client continuously yells, "It's 1999 and you are going to hurt me!" What action should the nurse do first?

Reorient the patient.

A client has undergone surgery for a spinal cord tumor that was located in cervical area. The nurse would be especially alert for which of the following?

Respiratory dysfunction

Which interventions are appropriate for a patient with increased ICP? Select all that apply. a) Administering prescribed antipyretics b) Frequent oral care c) Elevating the head of the bed at 90 degrees d) Encouraging deep breathing and coughing every 2 hours e) Maintaining aseptic technique with the intraventricular catheter

Which interventions are appropriate for a patient with increased ICP? Select all that apply. You selected: • Administering prescribed antipyretics • Frequent oral care • Maintaining aseptic technique with the intraventricular catheter Correct Explanation: Controlling a fever is an important intervention for a patient with increased ICP because fevers can cause an increase in cerebral metabolism and can lead to cerebral edema. Antipyretics are appropriate for control of fevers. It is imperative that the nurse use aseptic technique when caring for the intraventricular catheter because of its risk for infection. Oral care should be provided frequently because the patient is likely to be placed on a fluid restriction and will have dry mucous membranes. A nondrying oral rinse may be used. Coughing should be discouraged in a patient with increased ICP because it increases intrathoracic pressure, and thus ICP. Unless contraindicated, the head of the bed should be elevated at 30 to 45 degrees and in a neutral position to allow for venous drainage.

A 40-year-old client was admitted to the psychiatric unit after a suicide attempt in which the client was found standing on the edge of a bridge. Which statement made by the client would lead the nurse to suspect a potential imminent suicide attempt?

"How often does the night personnel make rounds?"

A nurse is planning discharge education for a patient who underwent a cervical diskectomy. What strategies would the nurse assess that would aid in planning discharge teaching? A) Care of the cervical collar B) Technique for performing neck ROM exercises C) Home assessment of ABGs D) Techniques for restoring nerve function

Ans: A Feedback: Prior to discharge, the nurse should assess the patient's use and care of the cervical collar. Neck ROM exercises would be contraindicated and ABGs cannot be assessed in the home. Nerve function is not compromised by a diskectomy.

A patient has just been diagnosed with Parkinson's disease and the nurse is planning the patient's subsequent care for the home setting. What nursing diagnosis should the nurse address when educating the patient's family? A) Risk for infection B) Impaired spontaneous ventilation C) Unilateral neglect D) Risk for injury

Ans: D Feedback: Individuals with Parkinson's disease face a significant risk for injury related to the effects of dyskinesia. Unilateral neglect is not characteristic of the disease, which affects both sides of the body. Parkinson's disease does not directly constitute a risk for infection or impaired respiration.

The nurse is preparing an elderly hospitalized client for discharge to home within the hour. What should be the priority for the nurse? Administer intravenous morphine for report of postoperative pain. Assess the need for pneumococcal and influenza vaccinations. Encourage physical activity of 30 minutes following breakfast daily. Instruct the client to receive at least 1 hour of sun exposure each day.

Assess the need for pneumococcal and influenza vaccinations.

The nurse caring for a patient with bacterial meningitis is administering dexamethasone (Decadron) that has been ordered as an adjunct to antibiotic therapy. When does the nurse know is the appropriate time to administer this medication? A. 1 hour after the antibiotic has infused and daily for 7 days B. 15 to 20 minutes before the first does of antibiotic and ever 6 hours for the next 4 days C. 2 hours prior to the administration of antibiotics for 7 days D. it can be administered every 6 hours for 10 days

B

When teaching a client about levodopa-carbidopa therapy for Parkinson's disease, a nurse should include which instruction? a) "Report any eye spasms." b) "Be aware that your urine may appear darker than usual." c) "Stop taking this drug when your symptoms disappear." d) "Take this medication at bedtime."

B) "Be aware that your urine may appear darker than usual." Explanation: Levodopa-carbidopa, used to replace insufficient dopamine in the client with Parkinson's disease, may cause harmless darkening of the urine. The drug doesn't cause eye spasms, although blurred vision is an expected adverse effect. The client should take levodopa-carbidopa shortly before meals, not at bedtime, and must continue to take it for life

A patient diagnosed with myasthenia gravis has been hospitalized to receive plasmapheresis for a myasthenic exacerbation. The nurse knows that the course of treatment for plasmapheresis in a patient with myasthenia gravis is what? A) Every day for 1 week B) Determined by the patient's response C) Alternate days for 10 days D) Determined by the patient's weight

B) Determined by the patient's response

The nurse is developing a plan of care for a patient newly diagnosed with Bell's palsy. The nurse's plan of care should address what characteristic manifestation of this disease? A) Tinnitus B) Facial paralysis C) Pain at the base of the tongue D) Diplopia

B) Facial paralysis

A nurse is planning the care of a 28-year-old woman hospitalized with a diagnosis of myasthenia gravis. What approach would be most appropriate for the care and scheduling of diagnostic procedures for this patient? A) All at one time, to provide a longer rest period B) Before meals, to stimulate her appetite C) In the morning, with frequent rest periods D) Before bedtime, to promote rest

C) In the morning, with frequent rest periods

Which nursing diagnosis takes the highest priority for a client with parkinsonian crisis? a) Imbalanced nutrition: less than body requirements b) Impaired urinary elimination c) Ineffective airway clearance d) Risk for injury

C) Ineffective airway clearance Explanation: In parkinsonian crisis, dopamine-related symptoms are severely exacerbated, virtually immobilizing the client. A client confined to bed during such a crisis is at risk for aspiration and pneumonia. Also, excessive drooling increases the risk of airway obstruction. Because of these concerns, the nursing diagnosis of an ineffective airway clearance takes the highest priority. Although imbalanced nutrition: less than body requirements, impaired urinary elimination, and risk for injury are also appropriate nursing diagnoses, they are not immediately life-threatening.

The critical care nurse is caring for 25-year-old man admitted to the ICU with a brain abscess. What is a priority nursing responsibility in the care of this patient? A) Maintaining the patient's functional independence B) Providing health education C) Monitoring neurologic status closely D) Promoting mobility

C) Monitoring neurologic status closely

The nurse is working in a long-term care facility. When assessing her patients, what body system dysfunction should the nurse look for as the leading cause of morbidity and mortality in the older adult population?

Cardiovascular

A client comes to the clinic for evaluation because of complaints of dizzinesss and difficulty walking. Further assessment reveals a staggering gait, marked muscle incoordination, and nystagmus. A brain tumor is suspected. Based on the client's assessment findings, the nurse would suspect that the tumor is located in which area of the brain?

Cerebellum

Which of the following outcomes would be most appropriate to include in the plan of care for a client diagnosed with a muscular dystrophy?

Client participates in activities of daily living using adaptive devices. pg. 2072

An elderly client reports fatigue with increased activity. He states that he walks 30 minutes five times each week. The nurse assesses the resting heart rate as 72 beats per minute; 10 minutes after walking, the client's heart rate is 102 beats per minute. The nurse instructs the client to Increase walking at a faster pace. Decrease walking frequency to three times each week. Continue to walk at his current level. Refrain from any form of exercise.

Continue to walk at his current level.

The nurse reviews the patient's drug regimen for treatment of a brain tumor. She explains to the patient why one of the following drugs would not be prescribed, even though it might have therapeutic benefits. Which drug would not be prescribed for this patient?

Coumadin

When caring for a client with myasthenia gravis who is receiving anticholinesterase drug therapy, the nurse must be able to distinguish cholinergic crisis from myasthenic crisis. Which of the following symptoms is not present in cholinergic crisis? a) Increased weakness. b) Diaphoresis. c) Increased salivation. d) Improved muscle strength after I.V. administration of edrophonium chloride.

D) Improved muscle strength after I.V. administration of edrophonium chloride. Explanation: Extreme muscle weakness is present in both cholinergic crisis and myasthenic crisis. In cholinergic crisis, I.V. edrophonium chloride, a cholinergic agent, does not improve muscle weakness; in myasthenic crisis, it does. Diaphoresis and increased salivation are not present in cholinergic crises

The nurse has asked the unlicensed assistive personnel (UAP) to ambulate a client with Parkinson's disease. The nurse observes the UAP pulling on the client's arms to get the client to walk forward. The nurse should: a) give the client a muscle relaxant. b) have the UAP keep a steady pull on the client to promote forward ambulation. c) assist the UAP with getting the client back in bed. d) explain how to overcome a freezing gait by telling the client to march in place.

D) explain how to overcome a freezing gait by telling the client to march in place. Explanation: Clients with Parkinson's disease may experience a freezing gait when they are unable to move forward. Instructing the client to march in place, step over lines in the flooring, or visualize stepping over a log allows them to move forward. It is important to ambulate the client and not keep them on bed rest. A muscle relaxant is not indicated.

A client states, "I'm worthless, and I don't deserve to live." This theme in the client's expressed thought may signal a maladaptive response to which disorder?

Depression

A client with Alzheimer disease becomes agitated while the nurse is attempting to take vital signs. What action by the nurse is most appropriate? A) Place the client in a secluded room until calm. B) Continue taking the vital signs. C) Distract the client with a familiar object or music. D) Document the inability to assess vital signs due to client's agitation.

Distract the client with a familiar object or music.

A nurse is providing care to a client with Parkinson's disease. The nurse understands the the client's signs and symptoms are related to a depletion of which of the following?

Dopamine

The client presents to the walk-in clinic with fever, nuchal rigidity, and headache. Which of the following assessment findings would be most significant in the diagnosis of this client? a) Change in level of consciousness b) Vector bites c) Seizures d) Vomiting

Vector bites Explanation: Possible exposure to mosquito bites can be beneficial in the diagnosing of encephalitis secondary to West Nile virus. Change in LOC, vomiting, and seizures are all symptoms of increased ICP and due not assist in the differentiating of cause, diagnosis, or establishing nursing care.

A patient with Huntington disease has just been admitted to a long-term care facility. The charge nurse is creating a care plan for this patient. Nutritional management for a patient with Huntington disease should be informed by what principle? A) The patient is likely to have an increased appetite. B) The patient is likely to required enzyme supplements. C) The patient will likely require a clear liquid diet. D) The patient will benefit from a low-protein diet.

Ans: A Feedback: Due to the continuous involuntary movements, patients will have a ravenous appetite. Despite this ravenous appetite, patients usually become emaciated and exhausted. As the disease progresses, patients experience difficulty in swallowing and thin liquids should be avoided. Protein will not be limited with this disease. Enzyme supplements are not normally required.

A patient with a new diagnosis of amyotrophic lateral sclerosis (ALS) is overwhelmed by his diagnosis and the known complications of the disease. How can the patient best make known his wishes for care as his disease progresses? A) Prepare an advance directive. B) Designate a most responsible physician (MRP) early in the course of the disease. C) Collaborate with representatives from the Amyotrophic Lateral Sclerosis Association. D) Ensure that witnesses are present when he provides instruction.

Ans: A Feedback: Patients with ALS are encouraged to complete an advance directive or "living will" to preserve their autonomy in decision making. None of the other listed actions constitutes a legally binding statement of end-of-life care.

An expected nursing intervention for a patient diagnosed with Bell's palsy would be which of the following? A) Applying a protective eye shield B) Encouraging the patient to eat on the affected side C) Avoiding analgesics D) Avoiding brushing of the teeth

Ans: A Feedback: Corneal irritation and ulceration may occur if the eye is unprotected. While paralysis lasts, the involved eye must be protected. The patient is encouraged to eat on the unaffected side, due to swallowing difficulties. Analgesics are used to control the facial pain. The patient should continue to provide self-care including oral hygiene.

A patient with a cervical disc herniation in the acute phase is complaining of numbness and tingling of his arms. What are the priority interventions for the nurse to perform? Select all that apply.

• Have the patient wear a cervical collar daily. • Provide NSAID therapy. During the acute phase, the nurse should assist the patient in wearing his or her cervical collar daily. The administration of NSAIDs will assist the patient to reduce pain and tissue swelling. Isometric exercises should not be started during the acute phase, and weight lifting exercises are never appropriate. Strengthening the legs will not assist in this condition and exercise should be very limited during the acute phase of this condition.

A patient with a brain tumor has begun to exhibit signs of cachexia. What subsequent assessment should the nurse prioritize? A) Assessment of peripheral nervous function B) Assessment of cranial nerve function C) Assessment of nutritional status D) Assessment of respiratory status

Ans: C Feedback: Cachexia is a wasting syndrome of weight loss, muscle atrophy, fatigue, weakness, and significant loss of appetite. Consequently, nutritional assessment is paramount.

The nurse would expect to find which of the following symptoms when assessing a 38-year-old patient diagnosed with multiple sclerosis? A) Vision changes B) Absent deep tendon reflexes C) Tremors at rest D) Flaccid muscles

Ans: A Feedback: Vision changes, such as diplopia, nystagmus, and blurred vision are symptoms of multiple sclerosis. Deep tendon reflexes may be increased or hyperactive, not absent. Babinski's sign may be positive. Tremors at rest aren't characteristic of multiple sclerosis; however, intentional tremors, or those occurring with purposeful voluntary movement, are common in patients with multiple sclerosis. Affected muscles are spastic rather than flaccid.

A patient is being admitted to the neurologic ICU with suspected herpes simplex virus encephalitis. What nursing action best addresses the patient's complaints of headache? A) Initiating a patient-controlled analgesia (PCA) of morphine sulfate B) Administering hydromorphone (Dilaudid) IV as needed C) Dimming the lights and reducing stimulation D) Distracting the patient with activity

C) Dimming the lights and reducing stimulation

An elderly client reports to the nurse that food does not taste right. During a dietary assessment, the nurse learns that the client uses a lot of sugar and salt. The client states, "It makes the food taste better." The nurse intervenes by... A) Encouraging the client to use lemon, spices, and herbs in the diet B) Increasing the amount of hot foods served to the client C) Bringing 8 ounces of the client's favorite drink to the client D) Instructing the client to eat small, frequent meals

Encouraging the client to use lemon, spices, and herbs in the diet

The nursing instructor is teaching the senior nursing class about neuromuscular disorders. When talking about Multiple Sclerosis (MS) what diagnostic finding would the instructor list as being confirmatory of a diagnosis of MS? a) Oligoclonal bands b) IV administration of edrophonium c) An elevated acetylcholine receptor antibody titer d) Episodes of muscle fasciculations

Oligoclonal bands Correct Explanation: Electrophoresis of the CSF, a technique for electrically separating and identifying proteins, demonstrates abnormal immunoglobulin G bands, described as oligoclonal bands. An elevated acetylcholine receptor antibody titer and IV administration of edrophonium are diagnostic of Mysthenia Gravis. Episodes of muscle fasciculations are characteristic of ALS.

The school nurse notes a 6-year-old running across the playground with friends. The child stops in mid-stride, freezing for a few seconds. Then the child resumes their progress across the playground. The school nurse suspects what in this child? a) A tonic-clonic seizure b) A complex seizure c) A partial seizure d) An absence seizure

An absence seizure Correct Explanation: Absence seizures, formerly referred to as petit mal seizures, are more common in children. They are characterized by a brief loss of consciousness, during which physical activity ceases. The person stares blankly; the eyelids flutter; the lips move; and slight movement of the head, arms, and legs occurs. These seizures typically last for a few seconds, and the person seldom falls to the ground. Because of their brief duration and relative lack of prominent movements, these seizures often go unnoticed. People with absence seizures can have them many times a day. Partial, or focal, seizures begin in a specific area of the cerebral cortex. A generalized seizure involves the whole brain

A gerontologic nurse is advocating for diagnostic testing of an 81-year-old patient who is experiencing personality changes. The nurse is aware of what factor that is known to affect the diagnosis and treatment of brain tumors in older adults? A) The effects of brain tumors are often attributed to the cognitive effects of aging. B) Brain tumors in older adults do not normally produce focal effects. C) Older adults typically have numerous benign brain tumors by the eighth decade of life. D) Brain tumors cannot normally be treated in patient over age 75.

Ans: A Feedback: In older adult patients, early signs and symptoms of intracranial tumors can be easily overlooked or incorrectly attributed to cognitive and neurologic changes associated with normal aging. Brain tumors are not normally benign and they produce focal effects in all patients. Treatment options are not dependent primarily on age.

A patient has been admitted to the neurologic ICU with a diagnosis of a brain tumor. The patient is scheduled to have a tumor resection/removal in the morning. Which of the following assessment parameters should the nurse include in the initial assessment? A) Gag reflex B) Deep tendon reflexes C) Abdominal girth D) Hearing acuity

Ans: A Feedback: Preoperatively, the gag reflex and ability to swallow are evaluated. In patients with diminished gag response, care includes teaching the patient to direct food and fluids toward the unaffected side, having the patient sit upright to eat, offering a semisoft diet, and having suction readily available. Deep tendon reflexes, abdominal girth, and hearing acuity are less commonly affected by brain tumors and do not affect the risk for aspiration.

A family member of a patient diagnosed with Huntington disease calls you at the clinic. She is requesting help from the Huntington's Disease Society of America. What kind of help can this patient and family receive from this organization? Select all that apply. A) Information about this disease B) Referrals C) Public education D) Individual assessments E) Appraisals of research studies

Ans: A, B, C Feedback: The Huntington's Disease Society of America helps patients and families by providing information, referrals, family and public education, and support for research. It does not provide individual assessments or appraisals of individual research studies.

A patient with an inoperable brain tumor has been told that he has a short life expectancy. On what aspects of assessment and care should the home health nurse focus? Select all that apply. A) Pain control B) Management of treatment complications C) Interpretation of diagnostic tests D) Assistance with self-care E) Administration of treatments

Ans: A, B, D, E Feedback: Home care needs and interventions focus on four major areas: palliation of symptoms and pain control, assistance in self-care, control of treatment complications, and administration of specific forms of treatment, such as parenteral nutrition. Interpretation of diagnostic tests is normally beyond the purview of the nurse.

A patient diagnosed with a pituitary adenoma has arrived on the neurologic unit. When planning the patient's care, the nurse should be aware that the effects of the tumor will primarily depend on what variable? A) Whether the tumor utilizes aerobic or anaerobic respiration B) The specific hormones secreted by the tumor C) The patient's pre-existing health status D) Whether the tumor is primary or the result of metastasis

Ans: B Feedback: Functioning pituitary tumors can produce one or more hormones normally produced by the anterior pituitary and the effects of the tumor depend largely on the identity of these hormones. This variable is more significant than the patient's health status or whether the tumor is primary versus secondary. Anaerobic and aerobic respiration is not relevant.

A 25-year-old female patient with brain metastases is considering her life expectancy after her most recent meeting with her oncologist. Based on the fact that the patient is not receiving treatment for her brain metastases, what is the nurse's most appropriate action? A) Promoting the patient's functional status and ADLs B) Ensuring that the patient receives adequate palliative care C) Ensuring that the family does not tell the patient that her condition is terminal D) Promoting adherence to the prescribed medication regimen

Ans: B Feedback: Patients with intracerebral metastases who are not treated have a steady downhill course with a limited survival time, whereas those who are treated may survive for slightly longer periods, but for most cure is not possible. Palliative care is thus necessary. This is a priority over promotion of function and the family should not normally withhold information from the patient. Adherence to medications such as analgesics is important, but palliative care is a high priority.

A patient has been admitted to the neurologic unit for the treatment of a newly diagnosed brain tumor. The patient has just exhibited seizure activity for the first time. What is the nurse's priority response to this event? A) Identify the triggers that precipitated the seizure. B) Implement precautions to ensure the patient's safety. C) Teach the patient's family about the relationship between brain tumors and seizure activity. D) Ensure that the patient is housed in a private room.

Ans: B Feedback: Patients with seizures are carefully monitored and protected from injury. Patient safety is a priority over health education, even though this is appropriate and necessary. Specific triggers may or may not be evident; identifying these is not the highest priority. A private room is preferable, but not absolutely necessary.

The clinic nurse caring for a patient with Parkinson's disease notes that the patient has been taking levodopa and carbidopa (Sinemet) for 7 years. For what common side effect of Sinemet would the nurse assesses this patient? A) Pruritus B) Dyskinesia C) Lactose intolerance D) Diarrhea

Ans: B Feedback: Within 5 to 10 years of taking levodopa, most patients develop a response to the medication characterized by dyskinesia (abnormal involuntary movements). Another potential complication of long-term dopaminergic medication use is neuroleptic malignant syndrome characterized by severe rigidity, stupor, and hyperthermia. Side effects of long-term Sinemet therapy are not pruritus, lactose intolerance, or diarrhea.

The nurse responds to the call light of a patient who has had a cervical diskectomy earlier in the day. The patient states that she is having severe pain that had a sudden onset. What is the nurse's most appropriate action? A) Palpate the surgical site. B) Remove the dressing to assess the surgical site. C) Call the surgeon to report the patient's pain. D) Administer a dose of an NSAID.

Ans: C Feedback: If the patient experiences a sudden increase in pain, extrusion of the graft may have occurred, requiring reoperation. A sudden increase in pain should be promptly reported to the surgeon. Administration of an NSAID would be an insufficient response and the dressing should not be removed without an order. Palpation could cause further damage.

The nurse is writing a care plan for a patient with brain metastases. The nurse decides that an appropriate nursing diagnosis is "anxiety related to lack of control over the health circumstances." In establishing this plan of care for the patient, the nurse should include what intervention? A) The patient will receive antianxiety medications every 4 hours. B) The patient's family will be instructed on planning the patient's care. C) The patient will be encouraged to verbalize concerns related to the disease and its treatment. D) The patient will begin intensive therapy with the goal of distraction.

Ans: C Feedback: Patients need the opportunity to exercise some control over their situation. A sense of mastery can be gained as they learn to understand the disease and its treatment and how to deal with their feelings. Distraction and administering medications will not allow the patient to gain control over anxiety. Delegating planning to the family will not help the patient gain a sense of control and autonomy.

The nurse is caring for a patient newly diagnosed with a primary brain tumor. The patient asks the nurse where his tumor came from. What would be the nurse's best response? A) "Your tumor originated from somewhere outside the CNS." B) "Your tumor likely started out in one of your glands." C) "Your tumor originated from cells within your brain itself." D) "Your tumor is from nerve tissue somewhere in your body."

Ans: C Feedback: Primary brain tumors originate from cells and structures within the brain. Secondary brain tumors are metastatic tumors that originate somewhere else in the body. The scenario does not indicate that the patient's tumor is a pituitary tumor or a neuroma.

A patient who has been experiencing numerous episodes of unexplained headaches and vomiting has subsequently been referred for testing to rule out a brain tumor. What characteristic of the patient's vomiting is most consistent with a brain tumor? A) The patient's vomiting is accompanied by epistaxis. B) The patient's vomiting does not relieve his nausea. C) The patient's vomiting is unrelated to food intake. D) The patient's emesis is blood-tinged.

Ans: C Feedback: Vomiting is often unrelated to food intake if caused by a brain tumor. The presence or absence of blood is not related to the possible etiology and vomiting may or may not relieve the patient's nausea.

The nurse educator is discussing neoplasms with a group of recent graduates. The educator explains that the effects of neoplasms are caused by the compression and infiltration of normal tissue. The physiologic changes that result can cause what pathophysiologic events? Select all that apply. A) Intracranial hemorrhage B) Infection of cerebrospinal fluid C) Increased ICP D) Focal neurologic signs E) Altered pituitary function

Ans: C, D, E Feedback: The effects of neoplasms are caused by the compression and infiltration of tissue. A variety of physiologic changes result, causing any or all of the following pathophysiologic events: increased ICP and cerebral edema, seizure activity and focal neurologic signs, hydrocephalus, and altered pituitary function.

While assessing the patient at the beginning of the shift, the nurse inspects a surgical dressing covering the operative site after the patients' cervical diskectomy. The nurse notes that the drainage is 75% saturated with serosanguineous discharge. What is the nurse's most appropriate action? A) Page the physician and report this sign of infection. B) Reinforce the dressing and reassess in 1 to 2 hours. C) Reposition the patient to prevent further hemorrhage. D) Inform the surgeon of the possibility of a dural leak.

Ans: D Feedback: After a cervical diskectomy, the nurse will monitor the operative site and dressing covering this site. Serosanguineous drainage may indicate a dural leak. This constitutes a risk for meningitis, but is not a direct sign of infection. This should be reported to the surgeon, not just reinforced and observed.

The nurse is caring for a patient with Huntington disease who has been admitted to the hospital for treatment of malnutrition. What independent nursing action should be implemented in the patient's plan of care? A) Firmly redirect the patient's head when feeding. B) Administer phenothiazines after each meal as ordered. C) Encourage the patient to keep his or her feeding area clean. D) Apply deep, gentle pressure around the patient's mouth to aid swallowing.

Ans: D Feedback: Nursing interventions for a patient who has inadequate nutritional intake should include the following: Apply deep gentle pressure around the patient's mouth to assist with swallowing, and administer phenothiazines prior to the patient's meal as ordered. The nurse should disregard the mess of the feeding area and treat the person with dignity. Stiffness and turning away by the patient during feeding are uncontrollable choreiform movements and should not be interrupted.

When developing a plan of care for a patient with Guillain-Barre' syndrome, the nurse knows that which of the following nursing interventions would receive priority? A) Using the incentive spirometer as prescribed B) Maintaining the patient on bed rest C) Assisting the patient with activities of daily living D) Determining abnormalities of cognitive function

Ans: A Feedback: Impaired gas exchange would be the priority. Respiratory function can be maximized with incentive spirometry and chest physiotherapy. Nursing interventions aimed at enhancing physical mobility and preventing a deep vein thrombosis are utilized. Assisting the patient with activities of daily living is important but would not be the priority nursing intervention. Guillain-Barre' does not affect cognitive function or level of consciousness.

When assessing a patient with myasthenia gravis, the nurse would be correct in questioning the patient regarding which of the following clinical manifestations? A) Weakness associated with fatigue B) Headache that worsens at night C) Projectile vomiting without nausea D) Diaphoresis

Ans: A Feedback: Myasthenia gravis, an autoimmune disorder affecting the myoneural junction, is characterized by varying degrees of weakness of the voluntary muscles. Generalized weakness affects all the extremities and the intercostal muscles, resulting in varying decreasing vital capacity and respiratory failure. The other manifestations listed are not symptomatic of myasthenia gravis.

When examining a patient with Guillain-Barre' syndrome, the nurse would expect to assess which of the following clinical manifestations? A) Paresthesias of the hands and feet B) Hyperactive deep tendon reflexes C) Hypotension D) Descending weakness

Ans: A Feedback: Sensory symptoms of Guillain-Barre' include paresthesias of the hands and feet, and pain related to the demyelinization of sensory fibers. Other clinical manifestations include hyporeflexia and loss of deep tendon reflexes. A classic feature of Guillain-Barre' is ascending weakness.

The nurse teaching a patient with trigeminal neuralgia about factors that precipitate an attack would be correct in teaching him to avoid: A) Washing his face B) Exposing his skin to sunlight C) Using artificial tears D) Drinking liquids at room temperature

Ans: A Feedback: Washing the face should be avoided if possible due to the fact that this activity can trigger an attack of pain in a patient with trigeminal neuralgia. Exposing the skin to sunlight is not harmful to this patient. Using artificial tears and drinking liquids at room temperature are appropriate behaviors.

It is important to frequently monitor the patient with Guillain-Barre' syndrome when ascending paralysis is occurring. When assessing the patient for bulbar muscle weakness, the nurse should be alert to which of the following clinical manifestations? A) Decreased level of consciousness B) Inability to clear secretions C) Hypersensitivity of hands and feet D) Increased intracranial pressure

Ans: B Feedback: Bulbar muscle weakness related to demyelinization of the glossopharyngeal and vagus nerves results in an inability to swallow or clear secretions. Guillain-Barre' does not affect cognitive function or level of consciousness. Sensory symptoms include paresthesias of the hands and feet related to demyelinization of the sensory fibers. Guillain-Barre' does not cause increased intracranial pressure.

A nurse caring for a patient with possible bacterial meningitis in the ICU knows that which of the following assessment findings would be expected for a patient with bacterial meningitis? A) Pain upon ankle dorsiflexion of the foot B) Neck flexion produces flexion of knees and hips C) Inability to stand with eyes closed and arms extended without swaying D) Numbness and tingling in the lower extremities

Ans: B Feedback: Clinical manifestations of bacterial meningitis include positive Brudzinski's sign. Neck flexion producing flexion of knees and hips correlates with a positive Brudzinski's sign. Positive Homan's sign (pain upon dorsiflexion of the foot) and negative Romberg's sign (inability to stand with eyes closed and arms extended) are not expected assessment findings for the patient with bacterial meningitis. Peripheral neuropathy manifests as numbness and tingling in the lower extremities and is not an initial assessment to rule out bacterial meningitis.

When teaching the patient with multiple sclerosis how to reduce fatigue, the nurse should tell him to: A) Take a hot bath. B) Rest in an air-conditioned room. C) Increase the dose of muscle relaxants. D) Avoid naps during the day.

Ans: B Feedback: Fatigue is a common symptom in patients with multiple sclerosis. Lowering the body temperature by resting in an air-conditioned room may relieve fatigue; however, extreme cold should be avoided. A hot bath or shower can increase body temperature, producing fatigue. Muscle relaxants, prescribed to reduce spasticity, can cause drowsiness and fatigue. Planning for frequent rest periods and naps can relieve fatigue. Other measures to reduce fatigue in the patient with multiple sclerosis include treating depression, using occupational therapy to learn energy conservation techniques, and reducing spasticity.

Which of the following clinical manifestations would alert the nurse caring for a patient with Guillain-Barré syndrome that his status is deteriorating? A) Tidal volume of 500 mL B) Residual lung volume of 1200 mL C) Vital capacity of 11 mL/kg D) Oxygen saturation of 97%

Ans: C Feedback: A vital capacity of 12 to 15 mL/kg in a patient with Guillain-Barre' means that the patient's condition has deteriorated to the point that he may need to be mechanically ventilated. Thus, a vital capacity of 11 mL/kg is a warning. The tidal volume, residual lung volume, and oxygen saturation are within normal values. Breathing in a Guillain Barre' patient would become increasingly labored as the paralysis ascended toward the intercostals and diaphragm.

The nurse knows that plasmapheresis is being utilized in the treatment of the patient with Guillain-Barre' syndrome for which of the following reasons? A) Removal of anti-acetylcholine receptor antibodies B) Reduction in the number of bacteria in the bloodstream C) Decrease in antibodies attacking peripheral nerve myelin D) Removal of potassium and fluid

Ans: C Chapter: 64 Cognitive Level: Application Difficulty: Moderate Integrated Process: Nursing Process Objective: 2 Patient Needs: A-1 Feedback: Plasmapheresis and IV immunoglobulin (IVIG) are used to directly affect the peripheral nerve myelin antibody level. Both therapies decrease circulating antibody levels and reduce the amount of time the patient is immobilized and dependent on mechanical ventilation. In myasthenia gravis, plasmapheresis is used to remove anti-acetylcholine receptor antibodies. Antibiotics reduce the number of bacteria in the bloodstream. Hemodialysis removes fluid and potassium.

The nurse recognizes that corticosteroid therapy, when used in the treatment of Guillain-Barre' syndrome, reduces the inflammation and edema associated with this neuromuscular disorder. It is most important for the nurse to monitor which of the following lab values for the patient on corticosteroid therapy? A) pH of urine B) Hemoglobin C) Serum potassium D) Serum glucose

Ans: D Feedback: Corticosteroid therapy increases the blood glucose level. Corticosteroids have an effect on insulin and can produce symptoms related to glucose intolerance

The nurse is caring for a patient recently diagnosed with myasthenia gravis whose CT scan reveals an enlarged thymus gland. Which additional assessment data would be consistent with the diagnosis of myasthenia gravis? A) Decreased sensation in the hands and feet B) Incoordination of gait C) Facial numbness causing slurred speech D) Generalized weakness of the extremities

Ans: D Feedback: Generalized weakness affects all the extremities and the intercostal muscles, resulting in decreasing vital capacity and respiratory failure in the myasthenia gravis patient. Myasthenia gravis is purely a motor disorder with no effect on sensation or coordination.

The nurse teaching a patient recently diagnosed with myasthenia gravis should tell him that it is caused by: A) Genetic dysfunction B) Upper and lower motor neuron lesions C) Decreased conduction of impulses in an upper motor neuron lesion D) A lower motor neuron lesion

Ans: D Feedback: Myasthenia gravis is characterized by a weakness of muscles, especially in the face and throat, caused by a lower neuron lesion at the myoneural junction. It isn't a genetic disorder. Combined upper and lower neuron lesions generally occur as a result of spinal injuries. A lesion involving cranial nerves and their axons in the spinal cord causes decreased conduction of impulses at an upper motor neuron.

The nurse is caring for a recently diagnosed patient with myasthenia gravis whose CT scan reveals an enlarged thymus gland. Which additional assessment parameter should the nurse complete to confirm the diagnosis of myasthenia gravis? A) Passive range of motion of the neck B) Check of deep tendon reflexes C) Application of painful stimuli to legs D) Visual screening using the Snellen chart

Ans: D Feedback: Patients with myasthenia gravis commonly exhibit diplopia (double vision) and ptosis. Using the Snellen chart enables the nurse to assess both of these clinical manifestations. Performing passive range of motion on the neck indicates whether or not the patient has nuchal rigidity, which is a clinical manifestation of meningitis, not myasthenia gravis. Checking deep tendon reflexes is not specific to myasthenia gravis. Application of painful stimuli assesses level of consciousness but also is not specific to myasthenia gravis.

A 48-year-old patient has been diagnosed with trigeminal neuralgia following recent episodes of unilateral face pain. The nurse should recognize what implication of this diagnosis? A) The patient will likely require lifelong treatment with anticholinergic medications. B) The patient has a disproportionate risk of developing myasthenia gravis later in life. C) The patient needs to be assessed for MS. D) The disease is self-limiting and the patient will achieve pain relief over time.

C) The patient needs to be assessed for MS.

A health care team is involved in caring for a client with advanced Alzheimer's disease. During a team conference, a newly hired nurse indicates that she has never cared for a client with advanced Alzheimer's disease. Which key point about the disease should the charge nurse include when teaching this nurse?

Clients with Alzheimer's disease are at high risk for injury because of their impaired memory and poor judgment.

A client experiences loss of consciousness, tongue biting, and incontinence, along with tonic and clonic phases of seizure activity. The nurse should document this episode as which type of seizure? a) Absence b) Generalized c) Jacksonian d) Sensory

Generalized Correct Explanation: A generalized seizure causes generalized electrical abnormality in the brain. The client typically falls to the ground, losing consciousness. The body stiffens (tonic phase) and then alternates between episodes of muscle spasm and relaxation (clonic phase). Tongue biting, incontinence, labored breathing, apnea, and cyanosis may also occur. A Jacksonian seizure begins as a localized motor seizure. The client experiences a stiffening or jerking in one extremity, accompanied by a tingling sensation in the same area. Absence seizures occur most commonly in children. They usually begin with a brief change in the level of consciousness, signaled by blinking or rolling of the eyes, a blank stare, and slight mouth movements. Symptoms of a sensory seizure include hallucinations, flashing lights, tingling sensations, vertigo, déjà vu, and smelling a foul odor.

A patient 3 days postoperative from a craniotomy informs the nurse, "I feel something trickling down the back of my throat and I taste something salty." What priority intervention does the nurse initiate? a) Notify the physician of a possible cerebrospinal fluid leak. b) Give the patient some mouthwash to gargle with. c) Ask the patient to cough to observe the sputum color and consistency. d) Request an antihistamine for the postnasal drip.

Notify the physician of a possible cerebrospinal fluid leak. Correct Explanation: Any sudden discharge of fluid from a cranial incision is reported at once, because a large leak requires surgical repair. Attention should be paid to the patient who complains of a salty taste or "postnasal drip," because this can be caused by cerebrospinal fluid trickling down the throat.

Upon admission, the physician orders baclofen (Lioresal) for a patient diagnosed with multiple sclerosis. The nurse knows that which of the following is an expected outcome of this medication? A) Reduction in the appearance of new lesions on the MRI B) Decreased muscle spasms in the lower extremities C) Increased muscle strength in the upper extremities D) Limited severity and duration of exacerbations

Ans: B Feedback: Baclofen, a GABA agonist, is the medication of choice in treating spasms. It can be administered orally or by intrathecal injection. Avonex and Betaseron reduce the appearance of new lesions on the MRI. Anticholinesterase agents increase muscle strength in the upper extremities. Corticosteroids limit the severity and duration of exacerbations.

A patient with myasthenia gravis is in the hospital for treatment of pneumonia. The patient informs the nurse that it is very important to take pyridostigmine bromide (Mestinon) on time. The nurse gets busy and does not administer the medication until after breakfast. What outcome will the patient have related to this late dose?

The muscles will become fatigued and the patient will not be able to chew food or swallow pills.

A client with cerebral metastasis suddenly experiences a seizure for which phenytoin 10 mg/kg intravenously is ordered as an initial loading dose. The client weighs 165 pounds. How many milligrams of phenytoin should the client receive? Enter the number ONLY.

750

The nurse is caring for a client who is being treated in the emergency department for a panic attack. Which nursing intervention would be most appropriate?

Stay with the client, emphasizing that the client is safe and that the nurse will remain with the client.

Which of the following nursing actions is most important in caring for the client following lithotripsy?

Strain the urine carefully for stone fragments.

The nurse is conducting an admission assessment for a client with major depressive disorder. Which is the priority assessment for the nurse?

Suicide risk assessment

The nurse is assessing a 42-year-old client who is experiencing depression. The client's mother died by suicide 20 years ago. Which statement regarding this client's risk for suicide is correct?

The client has a greater risk for suicide than the general population.

A client has an exacerbation of multiple sclerosis. The physician orders dantrolene, 25 mg P.O. daily. Which assessment finding indicates the medication is effective? a) Increased ability to sleep b) Relief from pain c) Relief from constipation d) Reduced muscle spasticity

d) Reduced muscle spasticity Dantrolene reduces muscle spacticity. It doesn't increase the ability to sleep or relieve constipation or pain.

An age-related change associated with the cardiovascular system is... decreased blood pressure. thinner heart valves. increased compliance of heart muscle. decreased cardiac output.

decreased cardiac output

Which risk factors predispose a client to the development of kidney stones? Select all that apply.

immobilization.

A nurse is preparing a presentation for family members of clients who have been diagnosed with depression. When describing the family response to depression, which would the nurse include?

Depression in one family member affects the entire family.

The nurse is educating a patient with urolithiasis about preventive measures to avoid another occurrence. What should the patient be encouraged to do?

Increase fluid intake so that the patient can excrete 2,500 to 4,000 mL every day, which will help prevent additional stone formation.

A client has just been diagnosed with Parkinson's disease. The nurse is teaching the client and family about dietary issues related to this diagnosis. Which of the following are risks for this client? Select all that apply.

• Dysphagia • Choking • Constipation

A client has been recently diagnosed with depression and has just started taking an antidepressant medication. Which of the client's statements indicates an accurate understanding of this aspect of treatment?

"I understand that I probably won't feel much better for a couple of weeks after I start the drugs."

A client with Parkinson's disease is prescribed amantadine hydrochloride 100 mg twice a day. The pharmacy supplies amantadine syrup, because the client has a history of difficulty swallowing tablets. The label reads 50 mg/5 mL. How many milliliters would the nurse administer to the client for each dose? Enter the correct number ONLY.

10

A patient has been diagnosed with meningococcal meningitis at a community living home. When should prophylactic therapy begin for those who have had close contact with the patient? A. within 24 hours after exposure B. within 48 hours after exposure C. within 72 hours after exposure D. therapy is not necessary prophylactically and should only be used if the person develops symptoms

A

The nurse is assisting with administering a Tensilon test to a patient with ptosis. If the test is positive for myasthenia gravis, what outcome does the nurse know will occur? A. thirty seconds after administration, the facial weakness and ptosis will be relieved for approximately 5 minutes B. after administration of the medication, there will be no change in the status of the ptosis or facial weakness C. the patient will have recovery of symptoms for at least 24 hours after the administration of Tensilon D. eight hours after administration, the acetylcholinesterase begins to regenerate the available acetylecholine and will relieve symptoms

A

The nurse is caring for a patient admitted to the hospital with a brain abscess that developed from an untreated case of otitis media. What assessment data is a priority to alert the nurse to changes in intracranial pressure? A. level of consciousness B. peripheral pulses C. sensory perception D. crackles bilaterally

A

The nurse is caring for a patient who is hospitalized with an exacerbation of MS. To ensure the patient's safety, what nursing action should be performed? A) Ensure that suction apparatus is set up at the bedside. B) Pad the patient's bed rails. C) Maintain bed rest whenever possible. D) Provide several small meals each day.

A) Ensure that suction apparatus is set up at the bedside.

The nurse is planning discharge education for a patient with trigeminal neuralgia. The nurse knows to include information about factors that precipitate an attack. What would the nurse be correct in teaching the patient to avoid? A) Washing his face B) Exposing his skin to sunlight C) Using artificial tears D) Drinking large amounts of fluids

A) Washing his face

A college student goes to the infirmary with a fever, headache, and stiff neck. The nurse suspects the student may have meningitis and has the student transferred to the hospital. If the diagnosis is confirmed, what should the nurse institute for those who have been in contact with this student? (Select all that apply.) A. administration of rifampin (Rifadin) B. administration of ciproflaxocin hydrochloride (Cipro) C. administration of ceftriaxone sodium (Rocephin) D. amoxicillin (Amoxil) E. rofecoxib (Vioxx)

A, B, C

A 67-year-old firefighter now has a neurological defect and has been informed that he will be transferred to a nursing home because his son is unable to care for him at home. While receiving a bed bath, the patient yells at the nurse, "You don't know what you are doing!" What is the best reaction by the nurse?

Accept the patient's behavior and do not take it personally.

A client has a suspected diagnosis of bladder stones. Stones may form in the bladder or originate in the upper urinary tract and travel to and remain in the bladder. What are some signs and symptoms that this client may be experiencing? Select all that apply.

All choices are true.

A patient who was diagnosed with Parkinson's disease several months ago recently began treatment with levodopa-carbidopa. The patient and his family are excited that he has experienced significant symptom relief. The nurse should be aware of what implication of the patient's medication regimen? A) The patient is in a "honeymoon period" when adverse effects of levodopa-carbidopa are not yet evident. B) Benefits of levodopa-carbidopa do not peak until 6 to 9 months after the initiation of treatment. C) The patient's temporary improvement in status is likely unrelated to levodopa-carbidopa. D) Benefits of levodopa-carbidopa often diminish after 1 or 2 years of treatment.

Ans: D Feedback: The beneficial effects of levodopa therapy are most pronounced in the first year or two of treatment. Benefits begin to wane and adverse effects become more severe over time. However, a "honeymoon period" of treatment is not known.

An older adult has a score of 12 on the Geriatric Depression Scale (GDS). What action should the nurse complete first? Encourage the client to participate in exercise activities. Encourage the client to discuss feelings. Notify the physician. Assess for the potential for self-harm.

Assess for the potential for self-harm.

A client with multiple sclerosis (MS) lives with her daughter and 3-year-old granddaughter. The daughter asks the nurse what she can do at home to help her mother. Which measure would be most beneficial? a) weekly visits by another person with MS b) regular exercise c) psychotherapy d) day care for the granddaughter

B) regular exercise Explanation: An individualized regular exercise program helps the client to relieve muscle spasms. The client can be trained to use unaffected muscles to promote coordination because MS is a progressive, debilitating condition. The data do not indicate that the client needs psychotherapy, day care for the granddaughter, or visits from other clients

In which location are most brain angiomas located?

Cerebellum pg. 2053

The nurse is describing hospice services to the family of a patient with end-stage heart failure. Which of the following would the nurse be least likely to include as a major focus of care? A) Invasive therapy B) Emotional support C) Pain control D) Symptom management

Invasive therapy

A nurse is assessing an elderly client with senile dementia. Which neurotransmitter condition is most likely to contribute to this client's cognitive changes? A)Increased norepinephrine level B)Decreased acetylcholine level C)Increased acetylcholine level D)Decreased norepinephrine level

Decreased acetylcholine level

A client with a traumatic brain injury has already displayed early signs of increasing intracranial pressure (ICP). Which of the following would be considered late signs of increasing ICP? a) Loss of gag reflex and mental confusion b) Mental confusion and pupillary changes c) Complaints of headache and lack of pupillary response d) Decerebrate posturing and loss of corneal reflex

Decerebrate posturing and loss of corneal reflex Correct Explanation: Early indications of increasing ICP include disorientation, restlessness, increased respiratory effort, mental confusion, pupillary changes, weakness on onside of the body or in one extremity, and constant, worsening headache. Later indications of increasing ICP include decreasing level of consciousness until client is comatose, decreased or erratic pulse and respiratory rate, increased blood presure and temperature, widened pulse pressure, Chenyne-Stokes breathing, projectile vomiting, heimplegia or decorticate or decerebrate posturing, and loss of brain stem reflexes (pupillary, corneal, gag, and swallowing).

The reason that federal and state governments carefully regulate treatment given in licensed health care facilities, particularly long-term care facilities, is expressed by which statement? A) Clients lack different perspectives. B) Clients are unable to make any health care decision. C) Clients lack capacity because of cognitive impairment. D) Older adult clients are vulnerable.

Older adult clients are vulnerable.

A client who recently experienced a stroke tells the nurse that he has double vision. Which nursing intervention is the most appropriate?

Alternatively patch one eye every 2 hours

A patient with possible bacterial meningitis is admitted to the ICU. What assessment finding would the nurse expect for a patient with this diagnosis? A) Pain upon ankle dorsiflexion of the foot B) Neck flexion produces flexion of knees and hips C) Inability to stand with eyes closed and arms extended without swaying D) Numbness and tingling in the lower extremities

B) Neck flexion produces flexion of knees and hips

Which findings indicate that a client has developed water intoxication secondary to treatment for diabetes insipidus? Tetany and increased blood urea nitrogen (BUN) levels Sunken eyeballs and spasticity Flaccidity and thirst Confusion and seizures

Confusion and seizures

You are caring for a 72-year-old client who has been admitted to your unit for a fluid volume imbalance. You know which of the following is the most common fluid imbalance in older adults? Hypovolemia Dehydration Hypervolemia Fluid volume excess

Dehydration

Which is the most common affective or mood disorder among older adults? Anxiety disorder Phobias Depression Schizophrenia

Depression

The nurse identifies a nursing diagnosis of imbalanced nutrition, less than body requirements related to difficulty in chewing and swallowing for a client with Parkinson's disease. Which of the following would be most appropriate for the nurse to integrate into the client's plan of care?

Encourage the client to massage the facial and neck muscles before eating.

The plan of care for a patient with advanced Alzheimer's disease includes the nursing diagnosis of risk for injury. The nurse has identified this nursing diagnosis most likely as related to which of the following?

Impaired memory

The plan of care for a patient with advanced Alzheimer's disease includes the nursing diagnosis of risk for injury. The nurse has identified this nursing diagnosis most likely as related to which of the following? Impaired memory Personality changes Separation from others Communication difficulties

Impaired memory

The nurse is called to attend to a patient having a seizure in the waiting area. What nursing care is provided for a patient who is experiencing a convulsive seizure? (Select all that apply.) a) Loosening constrictive clothing b) Providing for privacy c) Restraining the patient to avoid self injury d) Positioning the patient on his or her side with head flexed forward e) Opening the patient's jaw and inserting a mouth gag

• Loosening constrictive clothing • Positioning the patient on his or her side with head flexed forward • Providing for privacy Explanation: During a patient's seizure, the nurse should do the following. Loosen constrictive clothing. If possible, place the patient on one side with head flexed forward, which allows the tongue to fall forward and facilitates drainage of saliva and mucus. If suction is available, use it if necessary to clear secretions. Provide privacy, and protect the patient from curious onlookers. (The patient who has an aura [warning of an impending seizure] may have time to seek a safe, private place.) The nurse should not attempt to pry open jaws that are clenched in a spasm or attempt to insert anything. Broken teeth and injury to the lips and tongue may result from such an action. No attempt should be made to restrain the patient during the seizure, because muscular contractions are strong and restraint can produce injury.

A client has been scheduled to undergo a transurethral resection of the prostate (TURP) for benign prostatic hyperplasia (BPH). The nurse knows that the client understands preoperative teaching when he makes which statement?

"I understand I may develop urethral strictures as a result of having the TURP."

A nurse is completing an admission assessment of a young male client who has a history of depression and who was brought to the hospital by the client's partner. In response to the nurse's question regarding suicidal ideation, the client discloses that he is thinking about killing himself. Which question would be most appropriate for the nurse to ask next?

"What thoughts have you had about how you would kill yourself?"

A nurse is providing a fall prevention clinic for a group of older adults. What information should the nurse include? Select all that apply. A) Review medications routinely for side effects B) Place grab bars in the shower and tub C) Wear nonslip shoes or socks when walking D) Have routine vision and hearing screenings E) Place throw rugs on any un-carpeted surface

A) Review medications routinely for side effects B) Place grab bars in the shower and tub C) Wear nonslip shoes or socks when walking D) Have routine vision and hearing screenings

The nurse is teaching a patient with Guillain-Barré syndrome about the disease. The patient asks how he can ever recover if demyelination of his nerves is occurring. What would be the nurse's best response? A) "Guillain-Barré spares the Schwann cell, which allows for remyelination in the recovery phase of the disease." B) "In Guillain-Barré, Schwann cells replicate themselves before the disease destroys them, so remyelination is possible." C) "I know you understand that nerve cells do not remyelinate, so the physician is the best one to answer your question." D) "For some reason, in Guillain-Barré, Schwann cells become activated and take over the remyelination process."

A) "Guillain-Barré spares the Schwann cell, which allows for remyelination in the recovery phase of the disease."

A client arrives at the emergency department complaining of extreme muscle weakness after minimal effort. The physician suspects myasthenia gravis. Which drug will be used to test for this disease? a) Edrophonium b) Carbachol c) Pyridostigmine d) Ambenonium

A) Edrophonium Explanation: Edrophonium temporarily blocks the breakdown of acetylcholine, thus increasing acetylcholine level in the blood, and relieves weakness. Because of its short duration of action, edrophonium is the drug of choice for diagnosing myasthenia gravis. It's also used to differentiate myasthenia gravis from cholinergic toxicity. Ambenonium is used as an antimyasthenic. Pyridostigmine serves primarily as an adjunct in treating severe anticholinergic toxicity; it's also an antiglaucoma agent and a miotic. Carbachol reduces intraocular pressure during ophthalmologic procedures; topical carbachol is used to treat open-angle and closed-angle glaucoma

A patient with MS has been admitted to the hospital following an acute exacerbation. When planning the patient's care, the nurse addresses the need to enhance the patient's bladder control. What aspect of nursing care is most likely to meet this goal? A) Establish a timed voiding schedule. B) Avoid foods that change the pH of urine. C) Perform intermittent catheterization q6h. D) Administer anticholinergic drugs as ordered.

A) Establish a timed voiding schedule.

The nurse is developing a plan of care for a patient with Guillain-Barré syndrome. Which of the following interventions should the nurse prioritize for this patient? A) Using the incentive spirometer as prescribed B) Maintaining the patient on bed rest C) Providing aids to compensate for loss of vision D) Assessing frequently for loss of cognitive function

A) Using the incentive spirometer as prescribed

A clinic nurse is meeting with a group of older adults living in a community that has been experiencing extremely hot summer days. Which of the following measures would the nurse encourage the clients to practice to protect their health during the hot summer days? Select all answers that apply. A) Increase fluid intake. B) Decrease fluid intake C) Circulate air with a fan or air conditioner. D) Wear lightweight shirts and shorts. E) Decrease baths to 3 times per week. F) Take cool showers.

A) Increase fluid intake. C) Circulate air with a fan or air conditioner. D) Wear lightweight shirts and shorts. F) Take cool showers.

The nurse is working with a patient who is newly diagnosed with MS. What basic information should the nurse provide to the patient? A) MS is a progressive demyelinating disease of the nervous system. B) MS usually occurs more frequently in men. C) MS typically has an acute onset. D) MS is sometimes caused by a bacterial infection.

A) MS is a progressive demyelinating disease of the nervous system.

What assessment findings would the nurse expect to find with a client with progressive myasthenia gravis? a) Muscle weakness, difficulty swallowing, double vision, and difficulty speaking b) Muscle pain, difficulty speaking, headaches, and arthritic changes c) Muscle inflammation, choking when eating, nearsightedness, and painful joints d) Atrophy of the muscles, difficulty chewing, strabismus, and difficulty moving

A) Muscle weakness, difficulty swallowing, double vision, and difficulty speaking Explanation: With myasthenia gravis there is a disturbance in nerve transmission to the muscles. The signs and symptoms in this answer reflect this neuromuscular impairment. The other answers include signs and symptoms not related to neuromuscular impairment, such as atrophy, muscle inflammation, headaches, and arthritic changes.

The nurse is caring for a 77-year-old woman with MS. She states that she is very concerned about the progress of her disease and what the future holds. The nurse should know that elderly patients with MS are known to be particularly concerned about what variables? Select all that apply. A) Possible nursing home placement B) Pain associated with physical therapy C) Increasing disability D) Becoming a burden on the family E) Loss of appetite

A) Possible nursing home placement C) Increasing disability D) Becoming a burden on the family

A patient with Bell's palsy says to the nurse "It doesn't hurt anymore to touch my face. How am I going to get muscle tone back so I don't look like this anymore?" What interventions can the nurse suggest to the patient? A. suggest massaging the face several times daily, using a gentle upward motion, to maintain muscle tone B. suggest applying cool compresses on the face several times a day to tighten the muscle C. inform the patient that the muscle function will return as soon as the virus dissipates D. tell the patient to smile every 4 hours

A

The nurse is planning the care of a patient who has been recently diagnosed with a cerebellar tumor. Due to the location of this patient's tumor, the nurse should implement measures to prevent what complication? A) Falls B) Audio hallucinations C) Respiratory depression D) Labile BP

Ans: A Feedback: A cerebellar tumor causes dizziness, an ataxic or staggering gait with a tendency to fall toward the side of the lesion, and marked muscle incoordination. Because of this, the patient faces a high risk of falls. Hallucinations and unstable vital signs are not closely associated with cerebellar tumors.

The nurse would expect to document which of the following in a patient with myasthenia gravis undergoing a Tensilon test? A) Positive Tensilon test B) Negative Tensilon test C) Positive sweat test D) Negative sweat test

Ans: A Feedback: The patient in myasthenic crisis improves immediately following administration of edrophonium chloride (Tensilon). Sweat tests are used in diagnosing cystic fibrosis, not myasthenia gravis.

The nurse is caring for the patient with GBS in the intensive care unit and is assessing the patient for autonomic dysfunction. What interventions should be provided in order to determine the presence of autonomic dysfunction. A. assess the respiratory rate and oxygen saturation. B. assess the blood pressure and heart rate C. assess the peripheral pulses D. listen to the bowel sounds

B

To alleviate pain associated with trigeminal neuralgia, a patient is taking Tegretol (carbamazepine). What health education should the nurse provide to the patient before initiating this treatment? A) Concurrent use of calcium supplements is contraindicated. B) Blood levels of the drug must be monitored. C) The drug is likely to cause hyperactivity and agitation. D) Tegretol can cause tinnitus during the first few days of treatment.

B) Blood levels of the drug must be monitored.

A patient diagnosed with MS has been admitted to the medical unit for treatment of an MS exacerbation. Included in the admission orders is baclofen (Lioresal). What should the nurse identify as an expected outcome of this treatment? A) Reduction in the appearance of new lesions on the MRI B) Decreased muscle spasms in the lower extremities C) Increased muscle strength in the upper extremities D) Decreased severity and duration of exacerbations

B) Decreased muscle spasms in the lower extremities

When developing a long term care plan for the client with multiple sclerosis, the nurse should teach the client to prevent: a) fluid overload. b) contractures. c) dry mouth. d) ascites.

B) contractures. Explanation: Typical complications of multiple sclerosis include contractures, decubitus ulcers, and respiratory infections. Nursing care should be directed toward the goal of preventing these complications. Ascites, fluid overload, and dry mouth are not associated with multiple sclerosis.

When administering medications to an older adult patient, which medication does the nurse understand may remain in the body longer due to increased body fat? Diuretics Anticoagulants Barbiturates Digitalis glycosides

Barbiturates

A patient is diagnosed with a spinal cord tumor and has had a course of radiation and chemotherapy. Two months after the completion of the radiation, the patient complains of severe pain in the back. What is pain an indicator of in a patient with a spinal cord tumor?

Spinal metastasis pg. 2062

The nurse is assessing a client suffering from stress and anxiety. The most common physiologic response to stress and anxiety is:

gastrointestinal upset.


Kaugnay na mga set ng pag-aaral

What is the long term effect of the Crusades?

View Set

CH 14: Preoperative Nursing Management

View Set

White Fang Comprehension Chapters 4 - 7

View Set

Psychology Ch.1: Psychological Foundations

View Set

BIOD 121 Nutrition Class Module 2

View Set

Chapter 14: We Respond to God through Prayer

View Set

The Neurological System (part 2)

View Set

Digital Photography: Using Your Photos

View Set